Foundations of Reading (190) Practice Test

Lakukan tugas rumah & ujian kamu dengan baik sekarang menggunakan Quizwiz!

A sixth-grade teacher is planning an instructional unit on summarizing informational text. In one lesson, the teacher will show students how to use the graphic organizer shown below to support them in identifying important ideas from an informational passage. Topic of Passage: Main Idea 1 - 3 details Main Idea 2 - 3 details Main Idea 3 - 3 details Which of the following learning goals would be most essential for the teacher to address in the lesson to develop students' ability to compose an effective summary from these kinds of notes? A. citing evidence from the passage using direct quotes and paraphrasing B. developing a personal response to and interpretation of the passage C. noting unanswered questions that are raised in the passage D. analyzing the author's stylistic choices in the passage

Correct Response: A. A summary is a brief and succinct statement of the main points of a text. Option A is correct because identifying supporting evidence and using direct quotation and paraphrasing accurately in the graphic organizer will support students in developing an accurate summary. Accurately representing the relationships between main ideas and details is essential to comprehending the text and to preparing a coherent summary. Option B is incorrect because developing a personal response to a piece does not support the goal of writing a summary, which is to present succinctly the ideas or opinions of a source, quite distinct from one's own opinions. Options C and D are incorrect because, although an evaluation of unanswered questions (C) or an analysis of an author's style (D) may usefully follow a summary of a passage, these activities do not support the process of developing the summary.

A third-grade student performs below grade-level expectations in word-reading accuracy on informal assessments. Since the majority of the student's errors are with multisyllable words, the teacher plans to provide the student with daily explicit instruction for one week on the use of syllable-division strategies for reading multisyllable words. The teacher will then reassess the student at the end of the trial period. The primary benefit of this approach to informal assessment is that it: A. helps the teacher determine whether the student has the potential for improvement with short-term intervention. B. provides the teacher with nationally normed benchmarks with which to compare the student's progress. C. provides opportunities to engage a student who is not a skilled reader in authentic reading activities. D. contributes evidence that can be used to diagnose the student's specific reading disability.

Correct Response: A. Explicitly teaching the student common English syllable-division strategies, or patterns (e.g., VC/CV, V/CV, VC/V, V/V, /stable final syllable), will provide the student with an evidence-based strategy for dividing multisyllable words into decodable parts. By applying syllable-division strategies, the student would be more likely to accurately pronounce each syllable in an unfamiliar word, which would facilitate the student's ability to recognize it as a word in their oral vocabulary. Using an intervention strategy that has been proven to be effective in supporting readers' accurate decoding of multisyllable words will allow the teacher to determine if the student has the potential for improvement with a short-term intervention. If the retest results show no improvement, this would indicate to the teacher that the student likely lacks the foundational phonics skills necessary to benefit from instruction in syllabication. Option B is incorrect because teaching a particular strategy does not provide a teacher with national benchmarks. Option C is incorrect because the intervention described focuses on improving the student's word-level foundational reading skills development, not on text-level reading comprehension development. Option D is incorrect because most students require explicit instruction in syllabication strategies (and morphemic analysis) to support them in learning how to read unfamiliar multisyllable words accurately and automatically. Just because this student has not yet mastered reading multisyllable words does not suggest that the student has a specific reading disability.

Before beginning a new content-area reading passage, a fourth-grade teacher asks students to think of words related to the topic of the text. The teacher writes the words on the board and then asks the students to suggest ways to group the words based on meaningful connections. The teacher also encourages them to explain their reasons for grouping particular words together. This series of activities is likely to promote the students' reading development primarily by helping them: A. extend and reinforce their expressive and receptive vocabularies related to the text's topic. B. infer the meaning of new vocabulary in the text based on word derivations. C. strengthen and extend their understanding of the overall structure of the text. D. verify word meanings in the text by incorporating syntactic and semantic clues into their word analysis.

Correct Response: A. Grouping words related to a text based on conceptual categories and the words' associative meanings helps deepen students' understanding of the vocabulary. Discussing and justifying connections among the words further enhances students' understanding of the words and promotes retention of new words. Option B is incorrect because, while exploring the etymology of target vocabulary words can also be an effective strategy for deepening students' understanding of the words, the activities described in the scenario did not involve exploration of word derivations. Nor did they involve using semantic or syntactic clues from the text to verify words' meanings (D). Option C is incorrect because studying the conceptual connections among key vocabulary in a text is not likely to enhance students' understanding of the text's structure.

Use the information below to answer the three questions that follow. A third-grade teacher notes that students' vocabulary scores on a school-wide standardizedachievement test are below the national average and below their performance on the comprehension and decoding subtests. The teacher decides to collect ongoing data related to vocabulary instruction. The teacher records examples of students' use of new vocabulary during class discussions and has students turn in weekly examples of how they use new words in their writing. The teacher also has students maintain a vocabulary journal in which they develop definitions for new Tier Two and Tier Three vocabulary in their own words, make visual representations of word meanings, and analyze changes to a word's form and meaning when affixes are added. This approach to informal assessment benefits literacy instruction primarily by allowing the teacher to: A. monitor variou

Correct Response: A. In the scenario described in this item, the teacher collects data about students' vocabulary development across multiple language modes (e.g., listening, speaking, reading, and writing). In addition, analyzing students' use of vocabulary in multiple contexts allows the teacher to monitor students' vocabulary development in multiple dimensions, such as their depth of understanding of new vocabulary (e.g., as evidenced in their vocabulary journals) and their command, usage, and retention of new vocabulary (e.g., as evidenced during class discussions and in their daily writing). Option B is incorrect because the teacher does not use anecdotal or theoretical data to track students' progress. Option C is incorrect because the assessment strategies the teacher is using are not easily quantified. Option D is incorrect because the scenario only mentions evidence that the teacher is collecting with respect to students' vocabulary development and not each essential component of reading.

According to evidence-based practices, which of the following resources in a first-grade classroom best supports phonics instruction as a major component of reading instruction? A. decodable texts tied directly to the instructional scope and sequence of skills B. leveled texts considered to be at students' most accessible reading level C. predictable-patterned texts that are well supported by pictures D. themed alphabet books that accurately represent letter-sound relationships

Correct Response: A. Integrating decodable texts with phonics instruction aligns with evidence- based instruction. Practice applying newly learned phonics skills to read words is an essential component of an effective beginning reading program. Reading decodable texts gives beginning readers opportunities to practice applying newly learned phonics patterns and skills in the context of connected text to help build automaticity. Decodable texts aligned with the sequence of instruction in the reading curriculum emphasize words with phonics patterns previously taught. Because these texts feature regularly spelled words that match children's phonics knowledge, they encourage children to use decoding for word identification, which is prerequisite for developing accurate, automatic reading skills. Options B, C, and D are incorrect because leveled texts, illustrated predictable texts, and themed alphabet books do not limit words included in the text to those with known phonics patterns. These text types often use picture supports and repeated (i.e., predictable) phrases that promote the use of guessing as a word identification strategy, which undermines the development of strong decoding skills in beginning readers.

A group of fifth-grade students finishes reading a novel written in the first person. The teacher has the students work together to write a new account of a key scene as it might be reported by a different major character. Which of the following questions would be the most appropriate focus of a group discussion following the writing activity? A. How does the narrative point of view in a story shape a reader's understanding of events? B. How do key structural elements in a narrative work affect the emotional impact of a story? C. How does the author use figurative language to foreshadow the resolution in a narrative? D. How do descriptive and expository writing affect a reader differently in a narrative text?

Correct Response: A. This question focuses students' attention on a grade-level-appropriate literary analysis skill that directly builds on insights that would likely develop during the writing activity, such as the impact of narrative point of view on what is included in a story and on the tone of a story, and, in particular, how the narrative point of view (the vantage point from which the events in a story unfold) in the novel they read may have shaped their experience of the story as a reader. Options B, C, and D are incorrect because these questions are not related to the concerns and learning opportunities initiated in the writing activity.

A second-grade teacher frequently reads aloud informational books related to grade-level content in social studies, science, and the arts. The teacher supports students in developing their own questions during and after the read-alouds and then helps them conduct research on their questions using grade-level resources the teacher has collected on these topics. In keeping with evidence-based best practices, providing direct instruction in which of the following aspects of informational text would be most essential to students' success and growing independence in using informational texts for personal research? A. using text features to locate specific information in a text B. presenting information gained from a text in charts and tables C. using an outline to organize collected information in a logical format D. distinguishing between implied and explicitly stated information in a text

Correct Response: A. Learning to use text features builds students' ability to research questions independently. For example, reviewing a table of contents would support students in identifying what information is included in a book and in understanding how the information is organized by topic. Learning to use an index would help students locate specific information in the text. Options B and C are incorrect because these strategies focus on presenting the results of research in a systematic and clear way after research is complete. While good organization may shape ongoing research by prompting further questions, it does not provide students with the support they need to navigate informational texts independently. Option D is incorrect because distinguishing between implied and explicitly stated information is an advanced-level research skill more appropriate to an older student who has already mastered the fundamental features of informational texts.

A third-grade teacher periodically reads aloud from a chapter in content-area textbooks using think-aloud while reading. Following is an example. "The moon does not shine on its own. The sun's light reflects off the moon." Hmm. I'm imagining that the sun is like a flashlight shining on the moon in the dark. "As the moon rotates, only the part that faces the sun is visible from the Earth." I'm not quite sure what 'visible' means, but it sounds kind of like vision, which I know has to do with eyes. It probably means the part that we can see from the Earth. Now, that makes me wonder— why do we see different amounts of the moon at different times? Let's see if the next part of the chapter explains this. ..." The teacher's practice is most likely to promote students' reading comprehension of informational texts by: A. modeling for them metacognitive comprehension strategies. B. giving them an example of fluent oral read

Correct Response: A. Metacognitive reading comprehension strategies prompt students to think about their thinking as they read a text. The teacher models two powerful metacognitive strategies: visualizing to support comprehension and self-questioning to clarify understanding and to set a purpose for further reading. Option B is incorrect because the teacher pauses several times during reading to make comments about the text, so the teacher does not present an example of fluent oral reading. The teacher could conceivably summarize the text at the end of the think-aloud (C), but there is no evidence in the description to suggest this will occur. Option D is incorrect because, while the teacher talks about the vocabulary word visible and determines what it means deductively, the focus of this portion of the teacher's think-aloud is promot

A second-grade teacher pairs students with appropriate, accessible texts for a paired- reading activity. During the activity, two students sit side by side and take turns reading an entire short text aloud. Over a period of several days, the pairs of students read and reread a large number of accessible texts together. This activity best promotes students' development of: A. reading rate and automaticity. B. prosodic reading skills. C. comprehension skills and strategies. D. new phonics skills.

Correct Response: A. Option A is correct because it corresponds to evidence that indicates that rereading the same text several times can enhance comprehension of the text and can improve reading rate and automaticity with respect to the text, but that these gains do not necessarily transfer to other texts. The most effective way for students to improve reading rate and automaticity is to practice reading a wide range of texts that they can read accurately. Option B is incorrect because, while improving one's reading rate may improve one aspect of prosody (reading at an appropriate pace), it does not necessarily contribute to other aspects of prosody, such as reading with appropriate phrasing and expression. Option C is incorrect because the activity does not focus on developing students' comprehension skills and strategies. Option D is incorrect because the students are reading texts that are accessible and appropriate to them (i.e., texts they can read with a very high degree of accuracy and comprehension), so they are unlikely to encounter new phonics patterns in the texts.

A third-grade teacher has students work on their oral reading fluency each day using a repeated-reading approach. Students work with a classmate to take turns reading an assigned grade-level text and timing each other's oral reading fluency rate. Some students in the class are currently participating in Tier 2 interventions to address identified gaps in grade-level decoding skills. The teacher differentiates the repeated-reading activity for these students by selecting texts that are aligned with the decoding skills they have been studying. According to evidence-based best practices, which of the following additional modifications to the activity should the teacher make in order to improve the students' oral reading performance with their assigned text? A. providing the individual students with explicit teacher feedback with respect to their reading accuracy and prosody between readings B. increasing the amount of ti

Correct Response: A. Option A is correct because students who are learning, but have not yet mastered, the decoding skills necessary to read a particular text benefit from immediate corrective feedback as part of evidence-based practice designed to build their competence and independence applying these skills. Furthermore, the teacher's attention to the students' prosody provides them with explicit instruction in this aspect of fluency, while conveying to them the value of reading with expression. This is especially important because students who are not yet decoding automatically may tend to read passages word by word, or they may tend to read through punctuation to try to improve their rate at the expense of prosody and comprehension. Option B is incorrect because evidence suggests that reading the same passage up to three or four times provides the maximum benefit. Any additional readings are not likely to add value. Option C is incorrect because relying on context clues for word identification is a frequent cause of word-reading errors and the hallmark of a reader at risk for reading difficulty. Option D is incorrect because students should not engage in silent reading for fluency practice until their decoding is automatic. In addition, this strategy would not allow the teacher to monitor the students' reading accuracy or prosody.

A second grade teacher divides the class into small groups and provides a copy of the following instructions to each group. Students work collaboratively to complete the activity. Afterward, each group shares their results with the whole class during a teacher-led discussion. Look at the word fair in these two sentences: 1. It isn't fair that Juan got an extra scoop of ice cream 2: Simon and Mei-Yin went to the fair and rode on the merry-go-round. How are these words the same? How are they different? Can you think of sentences to show two different ways to use each of the words below? Saw. Bow. Root. Run. Play. Fly. Spell. Wind This activity and related discussion would best promote students' ability to: A. use semantic and syntactic clues to identify the meaning and pronunciation of homographs. B. apply structural clues to decode and identify the meaning of printed words. C. find and use synonyms fo

Correct Response: A. Option A is correct because the activity in the box promotes students' awareness of the characteristics that define a pair of homographs. Homographs are words that are spelled the same way but have different meanings. For example, the words saw, meaning the cutting tool, and saw, meaning the past tense of the verb to see, have different origins and meanings, but they are spelled the same way. Options B, C, and D are incorrect because drawing students' attention to the fact that two words can be spelled the same way but have completely different meanings would not directly contribute to their understanding of structural analysis (B), synonyms (C), or semantic analysis (D).

A group of first-grade students has mastered reading single-syllable words that follow the closed-syllable patterns VC, CVC, CVCC, and CCVC. The teacher would like to expand students' reading development by teaching them how to read two-syllable words that consist of closed syllables, such as picnic, muffin, trumpet, pretzel, invent, and frantic. The teacher could best promote the students' accurate, efficient reading of this type of word by teaching them to use which of the following decoding strategies? A. Look at the vowels in a target word; if they are separated by two consonants, divide the word between the consonants (e.g., muf/fin), and then read each syllable from left to right. B. Look for recognizable single-syllable words within a target word (e.g., muff in muffin, ant in frantic), and then use that word as a starting point to decode the longer word. C. Sound out each of the letters of a target word from

Correct Response: A. Option A is correct because the strategy is based on a common syllable-boundary pattern in English (VC/CV). For this reason, the strategy is effective with most two-syllable words containing two medial consonants with a vowel on either side. Options B, C, and D are incorrect because these decoding strategies are not consistent with the syllable structure of English words and therefore are not reliably accurate or efficient strategies for decoding multisyllable words. In addition, they do not build on the students' skill in recognizing single-syllable words that follow a closed-syllable pattern. Furthermore, Option D is incorrect because this strategy promotes guessing based on incomplete decoding, which would undermine the students' development of accurate decoding skill.

A teacher is selecting words to use to assess students' ability to segment the individual phonemes in spoken words. Which of the following words would require the highest level of skill with regard to orally segmenting phonemes? A. stamp B. catch C. fudge D. chase

Correct Response: A. Option A is correct because the word stamp is more challenging to segment than the words catch, fudge, and chase. The word stamp contains five phonemes, including blends in both syllable-initial and syllable-final positions. In particular, the two phonemes /m/ and /p/ in the final nasal blend -mp can be challenging for students to perceive and segment. Options B, C, and D are incorrect because, while the words may have complex spelling patterns, they each contain only three individual phonemes and no consonant blends: The word catch (B) contains the phonemes /k/, /ă/, and /ch/; fudge (C) contains the phonemes /f/, /ŭ/, and /j/; and chase (D) contains the phonemes /ch/, /ā/, and /s/.

Use the table below to answer the question that follows: Line TargetWord. Number of Speech Sounds 1 Though. 2 2. Best. 3 3. Fresh. 5 4. Scratch. 6 In which line in the table is a word accurately matched to the number of phonemes the word contains? A. Line 1 B. Line 2 C. Line 3 D. Line 4

Correct Response: A. Option A is correct because the word though is made up of two speech sounds: /th/ (spelled th) and /ō/ (spelled ough), so Line 1 of the chart is correct. B is incorrect because the word best has four distinct speech sounds, /b/, /ĕ/, /s/, and /t/, but the number of speech sounds appearing in Line 2 of the chart is 3. C is incorrect because the word fresh has 4 distinct speech sounds, /f/, /r/, /ĕ/, and /sh/, but the number of speech sounds appearing in Line 3 of the chart is 5. D is incorrect because the word scratch has 5 distinct speech sounds, /s/, /k/, /r/, /ă/, and /ch/, but the number of speech sounds appearing in Line 4 of the chart is 6.

Which of the following sentences contains a pair of italicized words that differ from each other by one phoneme? A: He took off his cap before he lay down to take a nap. B. She walked down the lane to see the beautiful deep blue sea. C. They were distracted by the noise as the window shade retracted. D. After he bashed his toe, he sat down for a while and basked in the sun.

Correct Response: A. Option A is correct because the words cap and nap differ only by their initial phoneme. A phoneme is a phonological unit of language—a discrete speech sound in a particular language that speakers of the language recognize as sufficient to distinguish between two phonologically similar but distinct words. For example, the speech sounds /k/ and /n/ are phonemes in English because native English speakers who hear the pair of spoken words cap and nap regard them as distinct words. Option B is incorrect because see and sea are pronounced the same way; that is, they contain an identical sequence of phonemes. Option C is incorrect because the two words differ by one morpheme (their prefix), but the prefixes differ by more than one phoneme. Option D is incorrect because bashed and basked differ in spelling by one letter, but they differ in pronunciation by two phonemes.

As students begin to read, the ability to blend phonemes orally contributes to their reading development primarily because it prepares students to: A: recognize high-frequency words in a text automatically B: combine letter-sounds to decode words C: guess the meaning of the unfamiliar words from their context D: divide written words into onsets and rime

Correct Response: B. Phonemic blending is the ability to combine a sequence of speech sounds (phonemes) together to form a word. Beginning readers use their skill in phonemic blending and their knowledge of letter-sound correspondences to sound out and blend the sounds of simple printed words. A, C, and D are incorrect because they describe literacy skills that are unrelated to phonemic blending.

An entering second-grade student performs well below benchmarks on the universal screening for oral reading fluency. These results are aligned with the teacher's observation that the student does not read with fluency when reading aloud during daily reading activities. At this stage of reading development, the factor that is most likely disrupting the student's reading fluency is that the student does not: A. have the phonics knowledge and skills needed to decode the words in the texts. B. know the meaning of most of the vocabulary words that appear in the texts. C. know how to deconstruct the complex language structures used in the texts. D. have sufficient background knowledge related to the texts' topics.

Correct Response: A. Option A is correct because, in the primary grades, the most common factor disrupting fluency is weak decoding skills. The key indicators of fluency are accuracy, rate, and prosody. In an oral reading fluency screening, a teacher uses a student's performance reading aloud a grade-level passage to take a quick measure of the students' overall reading development. Note that gaps in a student's phonics knowledge most directly affect reading accuracy, but inaccurate reading can also affect other fluency indicators (e.g., by causing a slow rate or choppy reading, by reducing a student's score in words correct per minute). Options B, C, and D are incorrect because the texts typically used in beginning-of-year, second-grade level screening assessments primarily feature words that represent the phonics skills and syllable types taught in first grade. Therefore, the complexity of the text would mostly likely not be at a level where vocabulary (B), language structure (C), or background knowledge (D) would be primary disruptors to oral reading fluency.

Which of the following tasks requires the most advanced level of skill along the phonological awareness continuum? A. orally segmenting the phonemes in the word chimp and then substituting /ŏ/ for /ĭ/ to make a new word, chomp B. orally segmenting the word wonderful into won/der/ful and then tapping the number of syllables in the word C. listening to the words place and pluck and then orally segmenting each word into its onset and rime D. listening to the words fiddle and fresh and then determining that both words begin with the same phoneme, /f/

Correct Response: A. Phonological and phonemic awareness skills develop along a continuum from basic to more complex skills. Phonemic awareness is a more advanced type of phonological awareness that involves the ability to distinguish and manipulate the individual phonemes in spoken words. Segmenting all the phonemes in a four-phoneme word (chimp) and then substituting the phoneme /ŏ/ for /ĭ/ to make a new word (chomp) are both tasks that involve complex skills at the higher end of the phonological awareness continuum. B, C, and D are incorrect because segmenting words into syllables and then counting the syllables (B), segmenting words into their onset and rime (C), and recognizing alliterative words—words that begin with the same phoneme (D)—are less complex skills that children develop earlier along the phonological awareness continuum.

Use the information below to answer the three questions that follow. A third-grade class includes students with delays in foundational reading skills. Two students also have delays in language expression and comprehension. The teacher is considering ways to best support the students' reading development. The teacher would also like to provide appropriate supports for the students during the planned biweekly whole-class close-reading routine, in which the teacher will engage the students in reading and rereading a variety of complex literary and informational passages. The teacher wants to support the students with literacy delays in developing self-confidence and self-efficacy as readers. Which of the following strategies would be most appropriate for this purpose? A. instructing the students explicitly in how to use various word-reading and comprehension-repair strategies to solve reading challenges B. requiring th

Correct Response: A. Providing the students with explicit instruction in the use of various word- reading skills and comprehension-repair strategies would allow them to access the more complex texts presented during whole-class reading routines. Options B and C are incorrect because they do not provide opportunities for the students to engage with complex grade-level texts. By using these strategies, the teacher would send the message that the teacher has low expectations regarding the students' capacity for growth in reading. This approach would not develop the students' feelings of self-efficacy and self-confidence as readers. It would more likely have the opposite effect. Option D is incorrect because simply exposing the students to different genres is not enough. The students in the scenario also need explicit instruction in skills and strategies for dealing with the challenges they will encounter in complex grade-level texts.

Some children in a kindergarten class have had limited prior exposure to storybooks read aloud and/or limited prior experiences discussing narrative texts. Their teacher wants to develop their knowledge of story structure. According to evidence-based best practices, which of the following instructional approaches is most likely to accelerate the children's understanding of the causal nature of story events? A. teaching story elements explicitly, such as main character, goal or problem, and resolution, as part of the daily read-aloud B. rereading favorite stories to children and letting them dress up like the characters C. having children practice putting photocopies of pictures taken from storybooks in the order in which they occurred in the book D. asking text-dependent questions about the key details in story events

Correct Response: A. Teaching children the elements of a story provides them with essential background knowledge needed for comprehension of narrative texts. Children with little experience hearing stories read aloud or participating in academic discussions of stories may lack familiarity with the conventions that govern story narratives (e.g., the events in a plot are generally all related to a central challenge or problem introduced at the beginning of the story; the events relate directly to the main character; the events frequently unfold as a sequence of actions and reversals until the challenge that set the story in motion is resolved). Evidence indicates that explicit instruction in these elements improves comprehension of literary texts. Option B is incorrect because, while this approach might increase children's interest and engagement in stories, it would not build the background knowledge needed to recognize the causal links between events in a story's plot. Option C is incorrect because, while this exercise may provide children with practice recalling major events in a story, it would not promote insight into the causal relationships between character and plot that create a unified story. Option D is incorrect because, while this approach may provide an effective assessment of children's recall of story events, it neither measures nor promotes their understanding of the causal relationships underlying the story's plot.

A kindergarten teacher encourages beginning readers to "write" their own captions beneath their drawings. This practice is most likely to lead to which of the following outcomes? A. The children's grasp of the alphabetic principle will be reinforced as they apply phonetic spelling. B. The children may become frustrated by the difficulty of the English spelling system and lose interest in writing. C. Because of the reciprocity between decoding and encoding, the children's reading progress may be adversely affected by any uncorrected spelling errors. D. The children will tend to develop automatic word-recognition skills by engaging in spelling practice.

Correct Response: A. The alphabetic principle is the understanding that letters represent the sounds (phonemes) of spoken language and that phonemes have a predictable, systematic relationship to letters and letter combinations. Evidence indicates that students' understanding of the alphabetic principle is reinforced and enhanced by having opportunities to apply their knowledge of letter-sound correspondences in both reading and writing—the latter by using phonetic spellings. Option B is incorrect because providing children with meaningful opportunities to write tends to be motivating rather than discouraging for beginning readers. Option C is incorrect because, at this stage of development, children would not be expected to spell words correctly but rather to apply their letter-sound knowledge to spell words phonetically, which promotes their reading progress. Option D is incorrect because children's phonetic spellings would not consistently match the conventional spellings used in texts, so engaging in this writing activity would not promote automatic word recognition.

Use the information below to answer the two questions that follow. When considering how to support students who are at risk for reading difficulties, an elementary school teacher first tries to align an individual student's profile with one of the following evidence-based reading-difficulty profiles. Profile 1: The student reads words accurately and efficiently but demonstrates needs in word knowledge and/or comprehension skills. Profile 2: The student demonstrates needs in decoding and word recognition but has strong word knowledge and comprehension skills. Profile 3: The student demonstrates needs in decoding and word recognition and also in word knowledge and comprehension skills. One advantage of the teacher keeping these general profiles in mind when considering whether a student may be at risk for reading difficulties is that it helps the teacher: A. determine a direction for specific diagnostic probes (e.g.,

Correct Response: A. The general profiles described in the scenario can help the teacher be more strategic and efficient when selecting appropriate diagnostic assessments for individual students. Gathering more information about an individual student's specific areas of difficulty will support the teacher in planning more targeted, effective interventions for the student. Option B is incorrect because a comprehensive intervention plan may not be warranted if a student's profile does not suggest that it is. For example, if a student reads grade-level passages accurately and fluently but cannot answer comprehension questions after reading, further diagnostic probing should focus on factors that affect comprehension rather than on all components of reading. Options C and D are incorrect because using these general profiles will not ensure that an intervention will integrate various language modes (C) or that the differentiation or intervention strategies the teacher uses will be standards based (D).

An entering third-grade student with a specific learning disability demonstrates reading comprehension that is below grade-level expectations. The student can read aloud narrative texts that are aligned with second-grade expectations with accuracy and fluency; however, the student does not consistently remember key details or events after reading the texts. In keeping with evidence-based best practices, which of the following strategies would be most appropriate for the teacher to try first to support the student's reading comprehension with literary texts? A. providing the student with explicit instruction in story elements using a graphic organizer B. engaging the student in repeated oral reading of grade-level texts with explicit teacher feedback C. providing the student with explicit instruction and guided practice in grade-level, Tier Three vocabulary D. having the student practice reading two- and three-sentenc

Correct Response: A. The information provided in the scenario indicates that the student reads aloud accurately and with fluency, but the student does not consistently remember key details or events after reading. The teacher should provide evidence-based instruction in narrative text structure (story elements) using a scaffold, such as a graphic organizer, as a first step in supporting the student's reading comprehension of literary texts. Graphic organizers based on story grammar would support the student in identifying and recalling key elements such as characters, setting, and events, as well as the relationships between story elements. The latter would also support the student's literary analysis. Option B is incorrect because this strategy is effective for developing fluency, which is not an area of difficulty for this student. Option C is incorrect because, while limited vocabulary knowledge can be a factor impeding reading comprehension, Tier Three vocabulary—technical vocabulary related to a specific discipline or content area—does not commonly appear in second-grade literary texts and, therefore, is unlikely to be the source of the student's comprehension difficulty. Option D is incorrect because this practice does not address the likely source of the student's difficulty, an inability to identify and recall important details in the context of a longer story. A three-sentence story would likely include only essential information, so it would not require discerning between important and unimportant details.

A teacher is planning reading instruction for a small group of students who exhibit the following characteristics. • The students can point accurately to words in predictable texts after listening to and following the teacher reading aloud and tracking the text several times using a big book format. • The students can identify the beginning and final sounds of simple three-phoneme words presented orally and can sometimes identify a word's medial sound. • The students have developed sound-symbols associations for the majority of consonant sounds and some vowel sounds. • The students can read several high-frequency words in simple texts. • The students can spell words with an accurate beginning consonant sound and sometimes an accurate final consonant sound. Given these characteristics, the students are most likely beginning to transition to which of the following phases of word reading? A. partial alphabeti

Correct Response: A. The students' profile in the scenario aligns with characteristics of students beginning to transition to the partial-alphabetic phase of word-reading development. Children in the pre-alphabetic phase can track and identify words in predictable text with rehearsal and have learned some high-frequency words by sight, as described in the first and fourth bullets. However, as children transition to the partial-alphabetic phase, they are beginning to make connections between their growing phonemic awareness and knowledge of letter-sound correspondences, so they begin to use some letter sounds to read and write words, usually focusing on the first letter sound in a word. The students in the scenario demonstrate these behaviors in the second, third, and fifth bullets. Option B is incorrect because the students do not attend to all the letter sounds in words, a key characteristic of the full-alphabetic phase. Since the students do not demonstrate behaviors characteristic of the full-alphabetic phase, they cannot possibly have the skills necessary to begin transitioning to more advanced phases (Options C and D).

Use the information below to answer the three questions that follow. A third-grade teacher notes that students' vocabulary scores on a school-wide standardized achievement test are below the national average and below their performance on the comprehension and decoding subtests. The type of reading assessment described in the scenario that compares students' reading performance to the performance of students in a national sample group can best be categorized as: A. summative. B. norm referenced. C. curriculum based. D. benchmark.

Correct Response: B. A norm-referenced reading assessment, a type of standardized assessment, compares students' performance with that of a norming group, which is a large sample of students typically at the same grade level or in the same age range. The score on a norm-referenced assessment is typically presented as a percentile ranking. Option A is incorrect because a summative assessment summarizes students' progress at the end of a time period after learning has taken place and does not compare students' performance against that of a norming group. Typical purposes of summative assessments include identifying what skills students have mastered during a specified time period or evaluating the effectiveness of a reading program or instructional strategy. Option C is incorrect because a curriculum-based reading assessment evaluates how well students have learned basic reading skills taught. The results can inform teachers of students' progress and challenges with respect to the reading curriculum. Option D is incorrect because benchmark assessments are interim assessments that track students' progress toward an instructional goal. Benchmark assessments can be used to help predict how well students are likely to perform on end-of-year state assessments.

A second-grade teacher works several times a week with a Tier 2 intervention group. At the beginning of the intervention, all the students in the group had strengths in oral reading fluency and challenges in text comprehension. As instruction proceeds, which of the following actions best aligns with key principles of a Multi-Tiered Systems of Support (MTSS) model of instruction? A. providing reading materials written at the most accessible level for the students B. adjusting instruction for individual students according to their responses to the intervention C. changing materials and resources frequently according to students' interests D. following a scripted intervention program verbatim with the students from beginning to end

Correct Response: B. Adjusting instruction for individual students according to their responses to intervention is a key principle of the Multi-Tiered Systems of Support (MTSS) model of instruction. The teacher should monitor the progress of the individual students in the Tier Two intervention group using an appropriate, evidence-based assessment, and then adjust instruction for individual students based on their responses to instruction. For example, some of the students might need more intensive instruction, while others may demonstrate mastery after a few lessons. Option A is incorrect because providing reading materials at the most accessible level will not provide opportunities for students to advance in their learning and to apply new skills. Option C is incorrect because instructional materials that are aligned with students' interests may not be aligned with their reading strengths and/or learning goals. Option D is incorrect because following a scripted program verbatim from beginning to end implies that the teacher is not attending to students' responses to the intervention and their changing strengths and needs.

A second-grade teacher is reading aloud a literary text to the class. Which of the following post-reading activities would be most likely to promote the students' comprehension of the story by enhancing their literary analysis skills? A. encouraging the students to identify the key vocabulary words in the story B. discussing with the students how the characters in the story respond to major events and challenges C. asking the students to reread the story silently and respond to several literal comprehension questions D. having the students freewrite about the story in their reading response journals

Correct Response: B. At the second-grade level, a significant focus of literary analysis centers on the characters in a story and how their actions and words can provide clues to their traits, motivations, and feelings. Options A, C, and D are incorrect because, although having students identify important vocabulary from the story (A), reread the story and respond to literal comprehension questions (C), or engage in a freewrite about the story (D) may enhance students' text comprehension, these activities would not develop their literary analysis skills.

Which of the following activities would be most effective for a teacher to use to promote students' strategic reading of literary texts? A. giving students bookmarks that list reading comprehension strategies with brief descriptions of their features and steps for their use B. leading students in frequent small-group discussions and close readings focused on relevant strategies (e.g., skimming, rereading) to use for different academic tasks and purposes C. reminding students to practice newly learned strategies (e.g., scanning for specific details) during independent reading and then enter the strategies they used in their reading logs D. pairing students with a classmate after a reading assignment and asking them to share which reading comprehension strategies they used

Correct Response: B. During small-group discussions and close reading lessons, the teacher can provide students with explicit explanation, teacher modeling, and guided practice of specific strategies to use with different literary texts and for various academic purposes. Evidence indicates that students apply reading strategies more effectively when they are explicitly taught when and how to use particular strategies. Options A, C, and D are incorrect because these approaches rely on the students having already achieved a certain level of mastery with the strategies. In addition, they do not provide an opportunity for the teacher to monitor students' performance and reteach strategies as needed.

Having kindergarten children practice tracing the letters of the alphabet in sand is most appropriate for children who need additional support in: A. internalizing the alphabetic principle and letter-sound correspondences. B. recognizing that print carries meaning. C. understanding the relationship between spoken and written language. D. developing letter-formation skills.

Correct Response: D. In the activity described, children use arm movements and highly textured material, which heighten the children's awareness of letter shapes and the sequence of strokes for forming letters. The activity is effective for students who are having difficulty with letter formation because it simultaneously activates visual, kinesthetic, and tactile sensory pathways in learning letter shapes and associating these shapes with their names. Evidence suggests that such multisensory techniques are effective for this purpose. Options A, B, and C are incorrect because the activity described does not address the relationship between the alphabetic writing system of English and the sounds of spoken language (A), the function of print (B), or oral language (C).

A prekindergarten teacher asks a small group of children to listen to and repeat what the teacher says. First, the teacher says the word mop and then pronounces it as /m/ and [ŏp].Next, the teacher says the word take and then pronounces it as /t/ and [āk]. This activity is likely to promote the children's phonological awareness primarily by: A. modeling how to separate the syllables in spoken words. B. showing them how to segment words into onsets and rimes. C. promoting their awareness of each phoneme in a spoken word. D. teaching them how to distinguish between consonants and vowels.

Correct Response: B. In the activity described, the teacher provides direct instruction in segmenting single-syllable words into onset and rime—that is, into the initial consonant sounds of the word (the onset) and the rest of the word (the rime). Promoting student mastery of onset-rime segmentation prepares students for learning phonemic awareness skills. Option A is incorrect because the segmenting activity described in the scenario focuses on single-syllable words, so it would not help students segment the syllables in multisyllable spoken words. Options C and D are incorrect because the segmenting activity described in the scenario does not focus on recognizing individual phonemes, so it would not promote the children's awareness of each distinct phoneme in a word (C) or their ability to distinguish between consonant and vowel phonemes (D).

Two proficient readers are answering post-reading comprehension questions about a chapter in a content-area textbook. • The first student demonstrates exceptional recall of details from the chapter but has difficulty answering questions about the gist of the chapter. • The second student can give an outstanding summary of the chapter but has difficulty remembering specific facts from the chapter. Which of the following best explains the most likely reason for the students' varied understanding of the text? A. The first student is more proficient than the second student at using metacognitive comprehension strategies to make sense of the text. B. Each student applied different reading comprehension skills when reading the text. C. The second student is more proficient at reading for literal understanding than for inferential understanding. D. Each student brought a unique set of prior experiences to the reading of

Correct Response: B. Many factors contribute to reading comprehension. Skilled readers use different comprehension strategies to achieve different purposes. For example, the first student described in this item may have skimmed the chapter to look for specific words or phrases mentioned in the comprehension questions, while the second student may have scanned the chapter for main ideas but did not dwell on individual terms or facts. Option A is incorrect because the first student had difficulty answering questions about the gist of the chapter, so it is likely that that student did not apply metacognitive strategies to enhance understanding during reading. Option C is incorrect because the second student was able to construct a strong summary of the chapter, which suggests an ability to use inferential skills. While background knowledge and experiences have a strong influence on reading comprehension (D), this factor cannot explain why the students focused on such different levels of the text.

A first-grade teacher would like to promote students' development of accurate decoding to support their oral reading fluency and reading comprehension. The teacher could most effectively promote first graders' accuracy by teaching them how to: A. use semantic and syntactic context clues in a text for word identification. B. apply phonics skills and knowledge of common syllable types and inflections to read words. C. memorize sets of grade-level words posted on classroom word walls by theme. D. sound out the first letter of a word and then guess the word based on a text's illustrations.

Correct Response: B. Option B is correct because accurate, fluent reading at the first-grade level depends on mastering basic decoding skills. Evidence indicates that teachers can support students' development of accurate decoding skills most effectively by promoting students' phonics knowledge and skills explicitly and systematically. In addition, teaching students the common English syllable types reinforces their phonics knowledge and builds orthographic knowledge that will support their accurate decoding of unfamiliar words they encounter in grade-level, decodable texts. Meanwhile, providing students with explicit instruction in common inflectional endings as part of phonics instruction expands their word-reading ability and allows them to read a greater range of decodable texts with accuracy and comprehension. Options A and C are incorrect because teaching students to rely on context clues for word identification (A) and teaching students to memorize words (C) do not contribute to students' development of phonics skills or orthographic knowledge, which are needed to support the development of accurate, automatic decoding. Accurate, automatic decoding is prerequisite for developing reading fluency that is sufficient to support reading comprehension. Option D is incorrect because guessing words based on incomplete decoding and partial letter-sound information would result in many word-reading errors. This would not support development of oral reading fluency or reading comprehension.

A second-grade teacher is working with a small group of students to improve their oral reading fluency. As part of lesson planning, the teacher analyzes the students' oral reading errors and plans instruction to address phonics knowledge and skills not yet mastered. The teacher's actions are likely to benefit the students' reading fluency most directly by: A. encouraging the students to slow down and decode words letter by letter, a prerequisite of fluent oral reading. B. improving the students' reading accuracy, a key component of fluency. C. promoting the students' recall of a large number of grade-level words by sight, a prerequisite of fluent oral reading. D. focusing the students on increasing their reading rate, a key component of fluency.

Correct Response: B. Option B is correct because providing individualized instruction based on students' assessed areas of need in phonics knowledge and skills will likely improve students' decoding accuracy. In an oral fluency assessment, accuracy is measured by the percentage of word-reading errors the student makes during oral reading. Option A is incorrect because second- grade level phonics instruction focuses on orthographic patterns larger than single letters. Promoting oral reading accuracy by encouraging students to read words letter by letter would be appropriate for students who are in an earlier phase of reading development, when they are still focused on learning letter-sound correspondence and blending skills. Options C and D are incorrect because these benefits do not correspond to the instruction described, which is focused on increasing the students' reading accuracy and not on developing their automaticity (C) or reading rate (D).

A second-grade teacher regularly reviews spelling patterns previously taught. The teacher also provides students with multiple opportunities to read and write connected text that features words containing the spelling patterns and to engage in word sorts comparing new and previously taught spelling patterns. These types of instructional activities are likely to promote students' reading skills primarily by developing their: A. skill in identifying and categorizing common word families by their rimes. B. accuracy and automaticity reading words that follow the target phonics patterns. C. automatic recognition of and fluency reading texts containing high-frequency words. D. knowledge of the meaning and usage of common, grade-level vocabulary words.

Correct Response: B. Option B is correct because reviewing previously introduced spelling patterns or phonics patterns, reading and writing words that feature those spelling/phonics patterns, and engaging in word sorts that compare new and previously taught spelling/phonics patterns not only contribute to students' development of spelling accuracy, but also promote their automaticity reading words that contain the target spelling/phonics patterns. For example, word sorts encourage students to look at every grapheme in printed words, which promotes their reliance on phonics skills for word recognition, a key factor in building accurate decoding skills. Options A, C, and D are incorrect because the activities described promote students' skill in categorizing word families (that is, words that share the same rime) (A) or reading high-frequency words (C), and they do not focus on developing students' knowledge of the meaning or usage of words (D).

A third-grade teacher is planning differentiated reading instruction for an entering-level English learner who has grade-level reading skills in a language that uses the Roman alphabet. The teacher could best accelerate the student's progress in reading English by using which of the following approaches? A. providing instruction to promote the student's development of the alphabetic principle by drawing explicit connections between oral and written language B. supporting the student in identifying consonant sounds that both languages have in common while systematically teaching common English syllable types to introduce English vowel patterns and pronunciations C. introducing systematic, explicit phonics skills to the student by showing the student how to sound out and blend the letter-sounds in simple English words D. modeling how to use textual and graphic context clues to decode unfamiliar words in texts while p

Correct Response: B. Option B is correct because the approach builds on the student's knowledge of reading in the home language, which can accelerate the student's progress in learning to read in English. The approach also includes explicit instruction in letter-sound relationships for English vowels, which are the phonemes most likely to differ between two languages that share the same alphabet. Introducing English vowel patterns and pronunciations by teaching common English syllable types is a systematic, explicit approach that aligns with evidence-based instructional practices for both English learners and students whose home language is English. Option A is incorrect because the student already has grade-level reading skills in an alphabetic language, which suggests that the student has developed insight into the alphabetic principle. Option C is incorrect because, at the third-grade reading level, the student will already be able to decode words in the home language, a skill that can transfer between alphabetic languages. Option D is incorrect, not only because this approach to word-identification is ineffective for promoting accurate decoding, but also because an entering-level English learner would not yet have the language skills to use context as a word-identification strategy in an English-language text.

A fifth-grade student reads the sentence, "After playing with her friends all day, Kaylee did her science homework, her geography project, and her English paper in one fell swoop." The student asks the teacher for support in understanding the meaning of the phrase one fell swoop. After explaining the phrase's meaning, the teacher could best deepen and extend the student's understanding of this idiomatic expression by: A. asking the student to find other sentences in the text that use the words fell and swoop. B. discussing with the student additional examples of the phrase used in context. C. directing the student to look up different meanings of fell and swoop in the dictionary. D. showing the student how to create a tree diagram of the structure of the phrase.

Correct Response: B. Option B is correct because the phrase one fell swoop is an idiom. An idiomatic expression is a word or set phrase that derives its meaning from common usage and not from the combined literal meanings of component words. After explaining the meaning of an unfamiliar idiom, a teacher can most effectively deepen a student's understanding of the idiom by providing several additional examples of the idiom used in comprehensible contexts. Options A and C are incorrect because one fell swoop functions semantically as a single unit, so strategies for determining the meaning of its component words would not help the student determine the meaning of the idiom as a whole. Tree diagrams (D) are used to represent the grammatical structure of a sentence. This would be useful in helping the student determine the grammatical function of the phrase in the sentence but not its meaning.

A fifth-grade teacher is about to begin a new unit on ecosystems, with an emphasis on the movement of matter among the various components of an ecosystem. Which of the following types of vocabulary words from the unit would be most appropriate for the teacher to pre-teach? A. words that are conceptually challenging B. high-frequency, phonetically irregular words C. high-frequency words that have multiple meanings D. multisyllable words made of two or more syllable types

Correct Response: B. Option B is correct because the phrase one fell swoop is an idiom. An idiomatic expression is a word or set phrase that derives its meaning from common usage and not from the combined literal meanings of component words. After explaining the meaning of an unfamiliar idiom, a teacher can most effectively deepen a student's understanding of the idiom by providing several additional examples of the idiom used in comprehensible contexts. Options A and C are incorrect because one fell swoop functions semantically as a single unit, so strategies for determining the meaning of its component words would not help the student determine the meaning of the idiom as a whole. Tree diagrams (D) are used to represent the grammatical structure of a sentence. This would be useful in helping the student determine the grammatical function of the phrase in the sentence but not its meaning.

Which of the following instructional strategies would be most effective in promoting students' decoding of multisyllable words that are not multimorphemic? A. giving students opportunities to read literature featuring predictable text containing multisyllable words B. teaching students how to divide multisyllable words into syllables according to common syllable types C. prompting students to sound out and blend the individual phonemes that compose multisyllable words D. developing and reinforcing students' recognition of high-frequency multisyllable words using flash cards

Correct Response: B. Option B is correct because the syllables in multisyllable words that are not multimorphemic (i.e., words that do not contain affixes) follow the same syllable types as those in single-syllable words (e.g., closed, open, r-controlled, vowel team). Thus, teaching students to recognize multisyllable words as a series of syllables allows them to apply their prior knowledge of syllable types to decode longer words accurately and efficiently. This strategy also supports them in applying syllable-division strategies to unfamiliar words. Option A is incorrect because reading predictable text is typically used with emergent readers and is not appropriate for teaching decoding of multisyllable words. Option C is incorrect because sounding out multisyllable words letter by letter is not an efficient strategy for decoding longer words. In addition, it does not encourage students to apply syllable-division strategies. Option D is incorrect because simply practicing memorizing high-frequency multisyllable words using flash cards does not provide students with a strategy for accurately decoding unfamiliar multisyllable words encountered in texts.

Use the table below to answer the question that follows: Line. Phonics Term. Example word 1 l-controlled vowel real 2 consonant trigraph scratch 3 vowel team stir 4 consonant blend mistake In which line in the table is the underlined portion of an example word accurately matched to the term that is used to describe that phonics element? A: Line 1 B: Line 2 C: Line 3 D: Line 4

Correct Response: B. Option B is correct because the underlined phonics element in the word scratch (tch) is a consonant trigraph. A consonant trigraph is a grapheme with three consonants that represents one phoneme (sound). The trigraph -tch is used to spell the /ch/ sound in the word scratch. Option A is incorrect because the underlined phonics element in the word real (eal) is not an l-controlled vowel. An l-controlled vowel is strongly colored by the sound /l/ (e.g., the vowel sound in the word pull) and is distinct from short and long vowel sounds frequently represented by the same vowel grapheme. Option C is incorrect because the phonics element underlined in the word stir (ir) is an r-controlled vowel, not a vowel team. A vowel team is a sequence of two or more letters that represent a single vowel sound (e.g., the letters oa in the word boat). Option D is incorrect because the phonics element underlined in the word mistake (st) is not a consonant blend in this context. The letter s is in the first syllable (the prefix mis-) and the letter t is in the second syllable of the word (the base word take). A consonant blend is a sequence of two or more consonants either in the beginning or end of a single syllable (e.g., the letter sequence st is a consonant blend in the words stop and last).

A second-grade teacher frequently conducts spelling inquiry workshops with students to deepen their understanding of various orthographic guidelines. For example, in one inquiry, the teacher gives students a set of words that follow the silent-e syllable pattern and that include the inflectional ending -ing, -ed, or -s. Students work with a classmate to sort the base words according to whether the word drops the silent e when adding the inflection. After sorting the words, the students must analyze each list to generate a rule describing the conditions that require an e-drop. The following is a completed chart by one pair of students. What is the rule for dropping silent e? Inflected words WITH an e-drop? Shaking, staring, baking, shared, biked, hoped, smiling, joked, living Inflected words WITHOUT an e-drop? shakes, stares, bakes, shares, bikes, hopes, smiles, jokes, lives Some pairs of students, such as the pai

Correct Response: B. Option B is correct because this strategy focuses students' attention on the phonics elements that affect the words' spelling: the final silent e in all of the base words and the initial letter of the inflections. When inflections begin with a vowel (i.e.,-ed, -ing), the silent e in the base word is dropped. In contrast, when the inflection begins with a consonant (i.e., the inflection -s) the e-drop rule does not apply. Options A, C, and D are incorrect because these strategies do not focus on a contrast between the two sets of words that could explain when an e-drop takes place. The sets of words on both sides of the chart have exactly the same morphological structure (A) and include exactly the same base words (C). Option D is incorrect because, while most of the words in the table contain a long-vowel sound, identifying the vowel-sound patterns of the words in the table would not help a student to generate the target spelling rule. The base word live (pronounced with short-vowel sound) is also an example of the target spelling pattern because it contains a final silent e which is dropped when adding the vowel suffix -ing.

A second-grade teacher wants to ensure that students become automatic in recognizing the orthographic patterns they are explicitly taught during decoding instruction. According to evidence-based best practices, which of the following instructional strategies best promotes students' automatic recognition of a new orthographic pattern to support proficient reading? A. providing instruction in the new orthographic pattern implicitly when it arises in the context of reading a shared text or it appears in a text selected for comprehension instruction B. providing practice with phoneme-grapheme mapping and various reading and spelling activities that focus on words containing the new orthographic pattern C. emphasizing the use of the three-cueing systems, especially context clues, to decode words that contain the new orthographic pattern D. emphasizing a tactile-kinesthetic approach when introducing new words that follow

Correct Response: B. Option B is correct because this strategy provides students with extensive practice focused on accurately encoding and decoding the newly learned orthographic patterns in word context. Evidence indicates that such practice strengthens the orthographic representation of the words in the students' mental lexicon and helps build automatic word recognition or automaticity, the ability to recognize words without conscious effort. Evidence also suggests that once new orthographic knowledge develops, a reader can more readily apply it to unfamiliar words in context. Option A is incorrect because evidence does not support implicit instruction as an effective method to achieve automatic word recognition. In addition, implicit instruction would not provide students with enough practice to develop automaticity. Option C is incorrect because scientific evidence indicates that this strategy, also known as the three-cueing systems, is the way poor readers attempt to read unfamiliar words in print; whereas, proficient readers use decoding to identify unfamiliar words. In addition, emphasizing context clues as a word-identification strategy distracts students from attending to the words' orthographic forms, which is necessary for developing automatic word recognition. The process of decoding or sounding out words is essential to a reader's acquisition of new orthographic knowledge, and the development of automaticity is essential for achieving sufficient fluency to support reading comprehension. Option D is incorrect because the strategy does not directly facilitate the decoding and encoding processes necessary for applying orthographic knowledge automatically.

Over the course of the school year, a sixth-grade student who had been a fluent, proficient reader in previous years has become increasingly inconsistent in comprehending grade- level literary and informational texts assigned in class. The results of informal, curriculum- based assessments indicate that the student still meets grade-level expectations in vocabulary knowledge, but the student's reading rate and comprehension have dropped below grade-level expectations. The teacher observes that the student does not read smoothly when reading aloud sentences that contain more than one clause, and the student often comments about "getting lost in the sentence." The teacher is also aware that the student tends to choose fiction and graphic novels for independent reading that are written well below grade-level expectations. The student's overall reading profile suggests that the student would likely benefit most from expl

Correct Response: B. Research has shown that independent reading of a wide range of literary and informational texts plays a key role in students' academic language development, as the language of books tends to be more sophisticated than that of everyday social language. The student in this scenario likely has adequate foundational reading skills, given the student's prior reading proficiency. However, the student's independent reading is limited to fiction and graphic novels written well below grade-level expectations, thus limiting the student's exposure to grade- level academic language. Explicit instruction focused on developing the student's command of grade-level academic language structures would support the student's continued progress in comprehending grade-level academic texts. Options A, C, and D are incorrect because the student had been a fluent, proficient reader in previous years and continues to read below-grade- level texts with pleasure. Thus, foundational reading skills are not likely causing the student's increasingly inconsistent reading comprehension of assigned academic texts.

Use the information below to answer the three questions that follow. A third-grade class includes students with delays in foundational reading skills. Two students also have delays in language expression and comprehension. The teacher is considering ways to best support the students' reading development. The teacher would also like to provide appropriate supports for the students during the planned biweekly whole-class close-reading routine, in which the teacher will engage the students in reading and rereading a variety of complex literary and informational passages. Which of the following guidelines for planning effective reading instruction for these students best addresses the evidence-based recommendation that foundational reading skills should be taught in conjunction with building a foundation for reading comprehension? A. providing sufficient time for the students to engage in sustained silent reading with se

Correct Response: B. Since most decodable texts are constructed specifically for the purpose of supporting students' decoding development, their stories are not necessarily sufficiently developed to support grade-level comprehension instruction, especially for older students such as the third graders in this scenario. A well-developed story includes, either explicitly or implied, essential story components or elements such as character(s), setting, an initiating event/problem, reactions to the initiating event/problem, attempts to carry out a plan in response to the main event/problem, consequences of the attempts or plan, and a resolution. Thus, by using decodable texts with an evidence-based scope and sequence for decoding instruction and complex, grade- level texts read aloud by the teacher for comprehension instruction, the teacher can support the students' ongoing development in both reading fluency and text comprehension. Options A, C, and D are incorrect because they do not explicitly address comprehension instruction.

Skimming is likely to be the most effective strategy for accomplishing which of the following reading tasks? A. evaluating the validity of information on an Internet website B. previewing a chapter in a content-area textbook C. synthesizing information from various sources for a research report D. studying specific facts for a content-area exam

Correct Response: B. Skimming involves a quick, superficial reading of a text to get an overall impression of the material. This would be an appropriate and effective strategy for previewing a textbook chapter. Options A, C, and D are incorrect because evaluating the validity of information on a website (A), synthesizing information from a variety of sources for research purposes (C), and studying facts for a content-area exam (D) all require a more careful reading of textual materials than skimming.

Considerations of validity in test construction relate most closely to: A. how a particular examinee's test performance relates to a preestablished standard. B. whether the test questions effectively measure their specified content. C. how a particular examinee's test performance compares to the performance of other examinees. D. whether the test results are likely to be repeatable with a similar examinee test group.

Correct Response: B. Test validity is the degree to which a test measures what it is intended to measure. Options A and C are incorrect because they describe how criterion-referenced tests (A) and norm-referenced tests (C) are scored. Option D is incorrect because it describes test reliability, which is a different concept related to test performance.

A kindergarten teacher is reading a big book to a group of children. The teacher periodically points to the beginning consonant of selected words and accentuates its initial phoneme as the teacher reads the word aloud. The teacher's practice is most likely to reinforce the children's: A. awareness of word boundaries in text. B. awareness of letter-sound correspondences. C. ability to segment the sounds of spoken words. D. ability to apply phonemic blending skills.

Correct Response: B. The activity described in the scenario contributes to students' beginning reading development by reinforcing their ability to distinguish the initial phoneme in a spoken word and to map that sound to print. By reinforcing letter-sound correspondences for the initial consonant in the target words, the teacher is promoting the children's understanding of the alphabetic principle. Option A is incorrect because the representation of word boundaries in text is not explicitly addressed in the activity. Option C is incorrect because, while the activity highlights the beginning consonant of words, the teacher does not segment the words into individual phonemes. Option D is incorrect because the teacher does not demonstrate phonemic blending skills in this activity.

Fourth-grade students silently read an assigned text for part of the English language arts block and then meet for a small-group discussion of the text in another part of the block. During the silent reading portion, the teacher instructs students to use stick-on notes to bookmark passages where they make predictions, ask or answer questions, or encounter an unfamiliar word. In addition to anchoring the group discussions in the text, this practice benefits students primarily by increasing their: A. silent reading fluency. B. metacognitive awareness while reading. C. level of reading enjoyment. D. ability to concentrate during reading.

Correct Response: B. The activity described in the scenario requires students to be aware of what they are thinking about as they are performing a mental activity (i.e., reading), the very definition of metacognitive awareness. To bookmark passages where they use comprehension strategies such as making predictions or self-questioning, or where they encounter an unfamiliar word, students must monitor their reading processes, including noting when comprehension breaks down. Options A, C, and D are incorrect because, although the activity described is likely to enhance students' reading comprehension over time by promoting their metacognition, in the short term, the activity interrupts their reading. As such, it is not specifically designed to promote their reading fluency (A), enjoyment of the text (C), or ability to concentrate on the text (D).

During a series of integrated science and literacy lessons, a third-grade teacher plans to have students read several articles from a children's magazine about new technologies for cleaning up pollution in the oceans. After they read the articles, students will work in small groups to create a graphic organizer comparing and contrasting two of the solutions. To prepare students to integrate information across texts in this activity, which of the following steps would be most essential for the teacher to take? A. providing a brief history of the environmental movement in the United States B. providing explicit instruction in how to identify the most important points and key details presented in the texts C. providing students with opportunities to share their personal responses to the articles D. providing explicit instruction in discipline-specific conventions of scientific writing (e.g., use of passive voice, quanti

Correct Response: B. The first step in developing the type of analysis described in the scenario is understanding important features or characteristics of each technology, and then identifying important areas of comparison and contrast between the two technologies to be compared. To do this, the students must be able to identify important points and key details in the articles because these are likely to focus on the practical considerations to be compared, such as effectiveness in solving the problem, cost, availability of materials, and time required to implement the solution. Options A, C, and D are not correct because these options do not promote background knowledge or skills relevant to the task at hand. Learning historical background information (A), developing a personal response to the articles (C), and becoming familiar with discipline-specific writing conventions (D) would not likely help students integrate and compare information about technological solutions to a problem.

A sixth-grade teacher is planning explicit instruction to develop students' ability to read and understand sentences that have a complex sentence structure. Which of the following skills would be most effective for the teacher to focus on? A. distinguishing between complex sentences that use passive and active voice B. deconstructing complex sentences into independent and dependent clauses C. identifying common transition words that link ideas in two or more sentences D. distinguishing between demonstrative and indefinite pronouns in a sentence

Correct Response: B. This skill would enable students to make sense of a complex sentence step by step. Complex sentences are sentences that contain two or more clauses: an independent clause, which is the main clause, and one or more dependent clauses, which support or clarify the meaning of the main clause. The challenge for students in understanding this sentence structure is in understanding how the component clauses work together to communicate a coherent idea. Deconstructing the sentence into its component clauses allows the student to understand the ideas in each part of the sentence separately before considering how the ideas in the clauses are related. Option A is incorrect because distinguishing between sentences written in active voice and passive voice does not address the essential difficulty in understanding sentences with multiple clauses. Options C and D are incorrect because identifying transition words that link separate sentences (C) or distinguishing between demonstrative and indefinite pronouns (D) would not develop students' ability to understand complex sentences.

Which of the following statements describes the most important reason for a fourth-grade teacher to assign a variety of high-quality trade books as a component of reading instruction? A. The themes typical of children's literature tend to reinforce students' development of literal comprehension skills. B. Reading across genres contributes to students' developing understanding of the structures and features of different texts. C. Simplified syntax and controlled vocabulary provide necessary scaffolding for students who are not skilled readers. D. Reading diverse texts promotes students' development of phonological and phonemic awareness skills.

Correct Response: B. Trade books are books marketed to a general audience. Children's trade books cover a range of topics and include both fiction and nonfiction texts. Wide reading of trade books provides students with exposure to many genres and helps them begin to develop an awareness of the structural differences between genres. Trade books are not limited to specific themes or lend themselves exclusively to literal comprehension (A), nor do they use simplified syntax or controlled vocabulary (C). Reading trade books does not explicitly (or implicitly) teach fourth-grade students phonological and phonemic awareness skills (D), which are typically developed at a much earlier grade level using primarily oral language activities.

Sixth-grade students have just finished reading a chapter in a novel and are getting ready to write an entry in their response journals. The teacher could most effectively develop students' literary response skills by assigning which of the following journal prompts? A. What new vocabulary words did you learn when reading this chapter? List and define the new words from the chapter. B. What happened in the chapter? Describe two or three events from the chapter. C. What do you think is the main idea or theme of the novel? Relate specific events in this chapter to the theme you suggest. D. Which characters are mentioned in this chapter? List each of the characters.

Correct Response: C. At the secondary level, formal literary response involves developing a thesis and providing evidence from the text to support the thesis. To help sixth graders learn how to construct an effective literary response to a text, the teacher should provide opportunities, such as prompted writing in response journals, to develop a thesis related to a text (e.g., identifying a main idea or theme of the text), and then to support the thesis by citing evidence from the text. Options A, B, and D are incorrect because these prompts do not target literary response skills. Option A emphasizes vocabulary development, while Options B and D focus students on literal comprehension skills only—that is, identifying factual information explicitly stated in the text.

Which of the following strategies would be most appropriate to use to promote second- grade students' ability to analyze key ideas and details in a literary text? A. explicitly teaching students the key features and conventions of different literary genres B. prompting students to evaluate the significance of a story's setting with respect to its theme C. asking students text-dependent who, what, where, when, why, and how questions about story elements D. encouraging students to clarify their understanding of a story by reflecting on their personal experiences

Correct Response: C. Characters, setting, plot, and a story's central lesson or message are important elements of story structure in narrative texts. At the second-grade level, students' literary analysis focuses primarily on these elements. Asking students direct, text-dependent wh- questions about the elements in a literary text would reinforce their understanding of these key components of story grammar as they are realized in a particular story, which supports the students' developing analysis of the text's key ideas and details. Option A is incorrect because learning about the features of different literary genres promotes reading comprehension in a general way but would not effectively support primary-grade students in learning to analyze key ideas and details in a particular text. Option B is incorrect because, while setting is a key story element, evaluating its significance with respect to the text's theme is a task that goes beyond grade-level expectations for most second-grade students. Option D is incorrect because it focuses more on students' personal responses to a text rather than their development of literary analysis skills.

According to basic principles of evidence-based, systematic phonics instruction, which of the following common English letter combinations would be most appropriate for a first-grade teacher to introduce first? A. ir B. kn C. th D. oi

Correct Response: C. In research-based, systematic phonics instruction, phonics elements are introduced according to their utility for beginning readers, and therefore according to their frequency of use in words appearing in primary-grades texts. Among the letter combinations given, th occurs most frequently in such texts and therefore is among the very first letter combinations taught. Options A, B, and D are incorrect because these letter combinations appear significantly less frequently in primary-grades texts than the letter combination th.

Use the information below to answer the three questions that follow. A third-grade teacher notes that students' vocabulary scores on a school-wide standardizedachievement test are below the national average and below their performance on the comprehension and decoding subtests. In response to students' performance on the vocabulary measure, the teacher plans to take a more systematic and robust approach to vocabulary instruction. The teacher's use of the data best underscores the importance of which of the following approaches to reading assessment? A. conducting both formal and informal reading-related assessments to plan instruction B. demonstrating knowledge of evidence-based strategies for assessing the major components of reading C. using results from reading assessments to guide instructional decision making D. determining students' current skills with respect to specific state grade-level standards

Correct Response: C. In the scenario described, the teacher makes an instructional decision—to make vocabulary instruction more systematic and robust—based on students' performance on the vocabulary subtest of the standardized schoolwide assessment. Their performance was below the national average and, more importantly, below their performance in other dimensions of reading measured by the assessment. Option A is incorrect because the scenario does not provide information about the students' performance on informal assessments. Options B and D are incorrect because the scenario only mentions the standardized schoolwide assessment and does not provide information about other assessment strategies the teacher is using (B) or whether the results on the standardized assessment provide information about how the students performed with respect to state standards (D). The scenario only mentions that the students performed "below the national average."

The teacher has arranged for various adult volunteers to participate during the morning "fluency warm-up." According to evidence-based instruction, the teacher could best use the volunteers to support students' development of prosody by: A. providing the volunteers with unfamiliar poems for individual students to practice reading aloud "cold" to a volunteer. B. having the volunteers time individual students' oral reading and then show the students how to record and track their own fluency data on a weekly basis. C. showing the volunteers how to model appropriate oral reading of a target poem and engage students in echo reading. D. asking the volunteers to monitor pairs of students to make sure both students are practicing a target poem and reminding them to take turns, if needed.

Correct Response: C. Option C is correct because an important component of evidence-based fluency instruction is providing students with explicit models of prosodic reading—reading that has the phrasing, pacing, and expression of spoken language. Echo reading is a strategy in which a fluent reader reads aloud a small portion of a shared text, and a student or students immediately reread the same short passage, echoing the model's performance. In Option C, the volunteer would provide an individual student with an explicit model of prosodic reading followed by scaffolded practice through the use of echo reading. Options A, B, and D are incorrect because having the volunteers listen to the students engage in a "cold" reading of a text (A), time the students' reading rate (B), or monitor the students' on-task behavior (D) would not provide the students with modeling of or instruction in prosodic reading.

Several children in a kindergarten class have mastered orally blending sets of spoken sounds together to make words. Which of the following additional skills demonstrated by the children would best indicate that they are ready to begin instruction in decoding simple words? A. identifying key parts of a book consistently, such as the front, back, and title, when prompted by the teacher and pointing to the first page B. tracking print directionality with a pointer on a big book of a predictable text after the teacher models reading and tracking the text C. identifying letter-sound correspondences consistently for several high-utility letters, such as a, m, t, and s, when prompted by the teacher D. recalling the letters of the alphabet in sequence when prompted by the teacher using an alphabet banner and the alphabet song

Correct Response: C. Option C is correct because knowledge of letter-sound correspondences is a necessary component of the more complex skill of decoding words. When early readers learn to decode simple words, they must learn to sound out the letters of a printed word in sequence and blend those sounds together to read the words. Mastering high-utility letters (i.e., letters that they are likely to encounter in beginning reading instruction and in grade-level decodable texts) prepares children to develop the complex skill of decoding. For example, the group of letters a, m, t, and s, includes both high-frequency consonants and a vowel, enabling a child who has mastered these letter-sounds to decode many simple, phonically regular words (e.g., sat, mat, at, am). Options A and B are incorrect because they describe foundational print concepts, book-handling skills (A) and print directionality in English (B), respectively. Without letter-sound knowledge, these more basic skills are not sufficient preparation to support beginning decoding instruction. Option D is incorrect because memorizing the letters of the alphabet in order is not a component skill of decoding and thus not an indicator of readiness for decoding instruction.

Which of the following rationales best describes the advantage of using poems for fluency practice? A. Poems frequently have predictable structures to support phonics development. B. Poetry resources are typically abundant in most classroom libraries. C. Poetry is meant to be read aloud and reread many times to construct meaning. D. Poems can be found in many lengths and address many topics.

Correct Response: C. Option C is correct because the explanation accurately reflects characteristics of poetry that support evidence-based strategies for promoting oral reading fluency. Evidence indicates that rereading or repeated reading is an effective strategy for improving oral reading fluency of texts. Reading poems provides an authentic context for rereading to promote prosody and comprehension. Poems are meant to be performed for an audience, and therefore rehearsal has an authentic purpose. Poems are also meant to be shared and can be read chorally or responsively by stanzas or lines. Option A is incorrect because this statement does not directly address fluency practice. Furthermore, encouraging students to rely on predictable text structures for word identification is not an evidence-based practice. Options B and D are incorrect because having poetry resources readily available (B) and having poems found in many lengths and about many topics (D) describe features that are not unique to poetry. In addition, these particular features do not relate directly to strategies for promoting fluency development.

A group of third-grade students reads a poem aloud accurately but without much expression. Before asking the students text-dependent questions about the poem's content, the teacher spends time focusing on phrase-cueing. For example, the teacher asks the students to "Read the phrase that tells us ________" or "Identify the phrase that describes ________ ". After focusing on key phrases, the teacher conducts an expressive oral reading of the poem, focusing on proper pausing and expression, especially with respect to the phrases they discussed. Finally, the teacher leads the students in an expressive choral reading of the poem. Engaging the students in these activities prior to discussing the meaning of the poem demonstrates the teacher's understanding of: A. the concept that poetry must be read aloud in order for readers to fully appreciate it. B. the importance of accuracy as the foundation of reading fluency and com

Correct Response: C. Option C is correct because the phrase-cueing activity described both highlights phrases that convey essential ideas in the poem and provides students with a context in which to speak about the phrases in a meaningful way. Recognizing component phrases in a written text is essential to reading the text with prosody—that is, reading with the natural phrasing and meaningful expression found in spoken language. Prosody is one of the three key indicators of reading fluency. Reading phrases expressively both supports and is supported by text comprehension. Options A, B, and D are incorrect because these options do not accurately reflect the lesson described. Reading aloud the poem (prior to analyzing its key phrases) does not result in appreciation of beauty or meaning in the poem (A). The phrase-cueing activities do not focus on developing students' decoding accuracy (B). Finally, the lesson focuses on only one key indicator of reading, prosody, and not on the other indicators, accuracy or reading rate (D).

Use the information below to answer the the question: A third-grade teacher uses word matrices as a strategy for teaching structural or morphemic analysis skills. The teacher models for students how to create and use a word matrix to generate related words by combining a base word or root with one or more affixes. Following a word matrix for the base move. re un move s ing ed er ment The teacher supports students in generating the following list of words using this matrix: moves, moving, moved, movement, movements, remove, removes, removing, removed, remover, unmoving, and unmoved Two students in the class with a learning disability are successful in generating multi morphemic words when presented with a familiar base word and a simple suffix (e.g., -s, -ed). However, when presented with a familiar prefix, base word or root, and suffix, they cannot generate any new words, with or without

Correct Response: C. Option C is correct because the strategy begins with what students already know (inflectional endings) and then increases the difficulty of the task incrementally, introducing just one derivational suffix at a time. In addition, the teacher also provides explicit explanation in how the meaning of an affix transforms the meaning of the base word. Differentiating the lesson in this way makes the task of generating multimorphemic words more accessible to the students without altering the lesson's learning objective. Option A is incorrect because this differentiation strategy increases the difficulty of the task by requiring more knowledge about roots and affixes than the original task. It also provides the students with less structure or scaffolding. Option B is incorrect because the strategy changes the lesson objective from learning morphemic analysis skills to learning spelling conventions related to affixes. Option D is incorrect because, by definition, nonsense words do not have meaning, an essential component of morphemic analysis.

A fourth-grade student tries to decode the word accumulate in a science article by using syllabication skills. In order to read the individual syllables of the word after dividing them correctly, the student would need to be proficient in decoding which of the following syllable types? A. silent e, closed, vowel team B. vowel team, consonant + le, silent e C. closed, open, silent e D. open, silent e, vowel team

Correct Response: C. Option C is correct because the word accumulate consists of the following syllable types: the first syllable of the word is closed; the second and third syllables are open, and the final syllable of the word is a silent-e syllable. Options A, B, and D are incorrect because the word accumulate does not contain a syllable with a vowel team. Furthermore, Option B is incorrect because the word accumulate does not contain a consonant + le syllable.

Use the information below to answer the two questions that follow. A fifth-grade teacher is planning a literature study focused on how various elements of an author's craft are used to convey a poem's theme. The class includes students with a wide range of reading and language skills, including several advanced readers. Over the course of the unit, the class reads and analyzes the themes of several, increasingly complex, grade-level poems. Near the end of the unit, the teacher will have students practice literary analysis skills on an unfamiliar grade-level poem. According to evidence-based best practices, which of the following differentiation strategies would be most appropriate for the teacher to use with advanced readers during this lesson? A. having advanced readers work independently at their own pace while other students work in small, heterogeneous groups to analyze a poem B. having all students work in smal

Correct Response: C. Option C is correct because this differentiation strategy provides all students, including advanced readers, with an appropriate level of challenge while learning grade- level skills. Having students work on this assignment in homogeneous small groups based on their skill level allows the teacher to use texts with varying levels of complexity to differentiate the task. Option A is incorrect because having advanced readers work independently while all the other students work collaboratively in small groups isolates the advanced readers and does not allow them to benefit from collaboration. Option B is incorrect because having advanced readers provide instructional support to other students does not provide the advanced readers with the challenge they require to deepen or expand their own reading skills. Option D is incorrect because having students who complete their work early analyze a second poem does not necessarily deepen or extend learning.

A first-grade teacher administers a spelling assessment midway through the school year. Afterward, the teacher analyzes students' spelling errors and categorizes the errors according to their most likely cause. Phonemic Awareness—The spelling error indicates difficulty perceiving all the sounds in words. Code—The spelling error indicates a code-based difficulty (i.e., mastery of specific phonics/morphemic elements and associated orthographic patterns). Several students in the class make spelling errors that primarily fall under the category of phonemic awareness. The students' spelling development would benefit most from an intervention focused on promoting their ability to apply which of the following foundational skills? A. identifying orally the onset and rime of a series of spoken words B. substituting target phonemes in spoken words to create new words C. segmenting sequentially all the phonemes that ma

Correct Response: C. Option C is correct because this intervention is appropriate for addressing a difficulty with phonemic awareness. Phonemic awareness is the ability to perceive individual speech sounds (phonemes) in words. At the first-grade level, spelling errors that fall under the phonemic awareness category most likely result from a student's incomplete perception of all the phonemes in a spoken word or the sequence of those phonemes. Instruction focused on sequentially segmenting all the phonemes in spoken words would enhance the students' accurate phoneme-grapheme mapping, or spelling. Options A, B, and D are incorrect because the phonological awareness skills targeted would not address the likely cause of the students' difficulty, which is perceiving the constituent sounds in a word in the correct sequence. Option A targets phonological units larger than the phoneme. The phoneme-substitution activity in Option B does not promote awareness of all the phonemes in a word or their sequence. Phoneme blending (D) is essential for supporting beginning decoding instruction but would not directly address the students' difficulty perceiving and sequencing all the phonemes in spoken words.

A prekindergarten teacher is reading a storybook to the class so that the children can see the words and pictures while the teacher points to the line of print. This activity best contributes to the children's emergent reading development primarily by: A. promoting their development of letter-recognition skills. B. helping them recognize the function of spaces between words. C. developing their awareness of left-to-right directionality. D. promoting their understanding of letter-sound correspondence.

Correct Response: C. Option C is correct because, by following the line of print with a finger or pointer while reading, the teacher provides students with a visual demonstration that print is read from left to right. Options A, B, and D are incorrect because in the scenario described the teacher does not explicitly identify letters (A), spaces between words (B), or letter-sound correspondences related to the text (D).

Students in a fourth-grade class read a text that includes the word indefensible, which is unfamiliar to some of them. Which of the following strategies for teaching the word would be most effective in both clarifying the meaning of the word and extending the students' understanding and use of an appropriate word-learning strategy? A. asking the students to paraphrase the sentence that contains the word by substituting a synonym for the word B. having the students enter the word in their ongoing list of new vocabulary words and then look up its definition independently C. modeling for the students how to apply knowledge of morphology to construct the word's meaning and use context to confirm it D. using print and digital reference materials to explain the word's meaning to the students before they read the text

Correct Response: C. Option C is correct because, by modeling how to use structural analysis and knowledge of familiar English morphemes to deduce the meaning of a new word (i.e., the root defense and the affixes in- and -ible), the teacher reinforces for students a powerful, independent word-learning strategy. Learning this strategy immediately extends students' understanding of the target word indefensible and, in the long term, promotes their ability to learn unfamiliar words that contain familiar morphemes. After using the strategy, the students should use context clues to verify that the meaning they have deduced makes sense in the text. Options A, B, and D are incorrect because they describe strategies that help students understand the meaning of a specific word, but they do not promote or extend the students' ongoing vocabulary development.

A first-grade teacher designs the following activity. 1. Divide students into pairs. 2. Have the two students sit back-to-back. 3. Give a student in each pair a picture of a familiar object to describe. 4. Have the second student try to name the object based on the description. 5. If the second student cannot determine the target object, instruct the student describing the object to use more precise language (e.g., describing the object's color, texture, size, use). 6. The student pairs switch roles and repeat steps 2-5. This activity is likely to contribute to students' literacy development primarily by: A. showing them how to distinguish shades of meaning among words with similar denotations. B. developing their ability to categorize words according to their function. C. promoting their oral language development and listening comprehension. D. encouraging them to develop their oral presentation skills.

Correct Response: C. Option C is correct because, in the activity described, the students must convey the identifying features of an object using oral language alone. This prompts the students to use descriptive language to try to evoke a mental image of the object in the minds of their partners. Conversely, the partners must listen attentively and focus carefully on these oral language clues to visualize and make accurate deductions about the objects. Options A and B are incorrect because the focus of the activity is on describing or interpreting descriptions of real objects using language. The activity does not focus on comparing words in terms of their connotative meanings (A) or syntactic category (B). Option D is incorrect because, while precision in language is one characteristic of an effective presentation, the setting in this activity is informal and conversational, and does not rely on or promote development of presentation skills.

Which of the following students is demonstrating the specific type of phonological awareness known as phonemic awareness? A. a student who, after being shown a letter of the alphabet, can orally identify its corresponding sound(s) B. a student who listens to the words sing, ring, fling, and hang and can identify that hang is different C. a student who, after hearing the word hat, can orally identify that it ends with the sound /t/ D. a student who listens to the word magazine and can determine that it contains three syllables

Correct Response: C. Phonemic awareness is the recognition that spoken words are made up of phonemes—the discrete speech sounds of a language. Identifying the final sound (or phoneme) in a word demonstrates phonemic awareness. Options A, B, and D are incorrect because the skills described, identifying the sound corresponding to a letter (A), recognizing rhyming words (B), and counting syllables in a word (D), can be performed without the ability to distinguish separate phonemes in a spoken word.

Use the information below to answer the two questions that follow. A fifth-grade teacher is planning a literature study focused on how various elements of an author's craft are used to convey a poem's theme. The class includes students with a wide range of reading and language skills, including several advanced readers. In an early lesson, the teacher distributes a simple short poem and a printed copy of the graphic organizer shown below. title/subject author/tone imagery figurative language The teacher has students read the poem twice. First, they read it silently. Then, they follow along as the teacher reads the poem aloud. In keeping with evidence-based best practices in reading instruction, which of the following steps would be most effective for the teacher to implement next in this lesson? A. asking students to complete the portions of the graphic organizer that they understand and then correcting any misunde

Correct Response: C. Providing explicit support and modeling how to complete the graphic organizer align with an evidence-based approach to reading instruction, in which a teacher provides direct explanation and explicit modeling before having students engage in guided practice and finally independent practice of a new reading skill. In this scenario, the teacher uses think-aloud and questioning to provide explanation and modeling of a literary analysis task. Options A, B, and D are incorrect because these strategies describe actions that the students are not yet adequately prepared to take because the strategies do not provide the students with sufficient instruction or support to complete the task.

A fifth-grade teacher gives students a "reading planner" for an informational text that they will be reading independently. The reading planner contains various activities, including prompting students to summarize certain passages, to explain relationships between concepts according to specific information in the text, and to determine the meaning of domain-specific words based on appositives or appositive phrases embedded in the text. This reading planner is likely to be most effective for achieving which of the following instructional purposes? A. developing students' ability to read the text evaluatively B. supporting students' development of prosodic reading skills C. encouraging students to read and interact closely with the text D. teaching students to adjust their reading rate based on text complexity

Correct Response: C. Reading planners such as the one described compel students to reread a complex academic text multiple times so that they can complete the activities in the planner, which are specifically designed to prompt the students to interact with the content and language of the text. Option A is incorrect because the activities do not focus on developing students' evaluative comprehension skills. Option B is incorrect because using a reading planner as described would not promote development of prosodic reading, since it is a silent reading activity that does not involve explicit or implicit teacher modeling of or instruction in prosodic reading. A text-based reading planner would not teach students how to adjust their reading rate based on text difficulty (D) because it focuses on just one specific text.

If a standardized test is said to lack reliability, the test: A. is not measuring what it is supposed to measure. B. has not proven to be useful as an instructional intervention. C. gives fluctuating scores in different administrations. D. has poor predictive value relative to students' classroom performance.

Correct Response: C. Reliability indicates the degree to which a test yields consistent results over successive administrations. If a test yielded fluctuating results, it would be considered to have low reliability. Options A and D are incorrect because they describe other dimensions of test performance (i.e., content validity [A] and predictive validity [D]). Option B is incorrect because standardized tests are not designed to serve as instructional interventions.

A fifth-grade class silently reads an informational text. In subsequent informal assessments, several students are able to read the text orally with fluency, but they demonstrate poor overall comprehension of the text. The teacher could most appropriately address these students' needs by adjusting future instruction in which of the following ways? A. using informational texts that are written at the students' independent reading level B. providing the students with explicit instruction in grade-level-appropriate test-taking strategies C. introducing a text's key vocabulary and supporting the students in close reading of key passages D. emphasizing reading skill-building activities that focus primarily on narrative texts

Correct Response: C. Since the students could read the text orally with fluency during subsequent informal assessments, the teacher needs to consider other factors that may have affected the students' comprehension of the text. Informational texts introduce vocabulary words that are likely to be unfamiliar to many students in a fifth-grade classroom. Informational texts may also include academic language structures with which students are unfamiliar. For students to be able to read and comprehend such a text independently, the teacher may need to model and provide students with practice in close reading of key passages in the text, as well as explicitly teach key vocabulary prior to reading. Using only informational texts that students can access independently (A) is not an effective option, as these texts are unlikely to include important grade- level vocabulary and thus may not match grade-level standards for that content area. Similarly, simply providing students with instruction in test-taking skills (B) or focusing on narrative texts for reading activities (D) would not help students develop the content-specific knowledge and vocabulary required at their grade level.

A sixth-grade student encounters the following sentence in a short story. She experienced a sense of Deja vu as she walked down the street of the strange new city. The student asks the teacher about the meaning of Deja vu in the sentence. The teacher could best respond by advising the student to take which of the following steps. A. making note of the word in a vocabulary log, and then studying the word after finishing the story B. using context clues in the sentence to guess the meaning of the word, and then trying out that meaning in the sentence C. looking up the word in the dictionary, and then paraphrasing the sentence using the dictionary definition D. breaking the word into its component parts, and then comparing the parts to the meanings of similar known words

Correct Response: C. Stopping to consult a dictionary is generally disruptive to the reading process; however, in some situations, as with the sentence shown in the box, consulting a dictionary is the only effective method for determining the meaning of an unfamiliar term encountered in a text. Also, paraphrasing the sentence by substituting the dictionary definition for the unfamiliar term is a good method for both reinforcing understanding of the new term and returning the reader to the flow of the story. The other strategies given would not be effective in this particular situation. Option B is incorrect because contextual analysis would not be a useful strategy here, since the sentence around the term déjà vu provides little or no clues as to the term's meaning. Using structural analysis (D) would also not be helpful, since the term is made up of two words and they are borrowed from the French language. Finally, simply noting the term in a vocabulary log and coming back to it after finishing reading the story (A) would cause the student to miss the significance of the sentence and could diminish the student's comprehension of the story.

A third-grade teacher has been conducting a series of ongoing assessments of a student's oral reading. Shown below is a sentence from a text, followed by a transcription of the student reading the text. The sample is representative of the student's typical oral reading performance. Text: Up ahead, Julia saw the overturned boat disappear over the waterfall's edge. Student: "Up ahead, Julie saw the overtired boat (pauses reading) overturned boat disappoint over the water's edge (pauses reading) disappear over the water's edge." Given the information provided, the teacher could best address this student's needs by providing targeted, evidence-based instruction focused on: A. developing the ability to self-monitor reading comprehension. B. enhancing oral vocabulary development and use of word-learning strategies. C. improving reading accuracy by attending to all parts of a word. D. promoting the ability to track connecte

Correct Response: C. Targeted instruction focused on enhancing the student's ability to attend to all parts of words during decoding is responsive to the typical errors the student makes in the assessment. In the representative sample, the student accurately reads most of the words. The student also reads the beginnings of longer, morphologically complex words accurately (i.e., correctly decoding the first morpheme in the compound word overturn and the complex word disappear) but misreads the remaining portion of both words. In each case, the student guesses the word based on its first syllable rather than decoding all parts of the word. Options A, B, and D are incorrect because they identify reading skills that the student clearly has already mastered, as evidenced in the representative sample. The student demonstrates comprehension monitoring by self-correcting in real time when word-reading errors do not make sense in the context (A). The student also demonstrates good comprehension and command of reading and oral vocabulary (B). Finally, the student demonstrates competence when tracking connected text by reading words in order across lines and from line to line (D) in the representative sample.

A second-grade class is studying a social studies unit focused on geography (e.g., bodies of water, landforms) and its effects on people. So far, the students have learned about lakes, oceans, and bays. As part of the unit, the teacher reads aloud an informational passage that explains why human settlements near rivers historically have succeeded and grown. The teacher pauses regularly to discuss the reading, using a range of text-based questions to prompt discussion and promote students' literal, inferential, and evaluative comprehension. Part of the text appears below. Rivers have fresh water. Fish and other wildlife live in and near rivers. The soil near rivers is good for growing crops and grazing animals. But rivers provided towns with more than just what people needed to survive. Moving water provided a source of power that people could harness for industry. And, over time, large rivers became superhighways fo

Correct Response: C. The answer to the question in Option C is not explicitly stated in the text. Therefore, to answer this question, the students need to make inferences based on information in the text and their own background knowledge about what people need to survive and thrive. Options A, B, and D are incorrect because the answers to these questions are explicitly stated in the text. Therefore, they only require literal comprehension.

Use the information below to answer the two questions that follow. When considering how to support students who are at risk for reading difficulties, an elementary school teacher first tries to align an individual student's profile with one of the following evidence-based reading-difficulty profiles. Profile 1: The student reads words accurately and efficiently but demonstrates needs in word knowledge and/or comprehension skills. Profile 2: The student demonstrates needs in decoding and word recognition but has strong word knowledge and comprehension skills. Profile 3: The student demonstrates needs in decoding and word recognition and also in word knowledge and comprehension skills. The teacher is planning progress monitoring for students whose reading performance fits one of the three profiles and who will be receiving differentiated instruction or an intervention to address their identified needs. Which of the fo

Correct Response: C. The primary purpose and benefit of progress-monitoring is to adjust instruction according to a student's response to the instruction. Progress-monitoring needs to occur frequently to be effective, so it is important that it captures even small improvements, which will help the teacher make appropriate adjustments to the content, intensity, or pace of the differentiated instruction or intervention. Option A is incorrect because monitoring students' stronger skills with the same frequency as the students' weaker skills would not be an efficient use of assessment because it would not necessarily result in information that can inform instruction in the students' area(s) of need. Option B is incorrect because standardized, norm- referenced assessments are not designed to be given frequently or show incremental progress. They are summative in nature and indicate how the students compare with other students in a particular area of reading. They do not provide information about how to adjust differentiated instruction or an intervention. Option D is incorrect because group-administered assessments would not present the same opportunities as individual assessments for the teacher to identify nuances of a student's performance that might result in adjustment of instruction. In addition, some skills assessments must be administered individually.

A fifth-grade teacher plans to have students read a chapter about the American Revolutionary War from their social studies textbook. The following is an excerpt from the chapter. The Battle of Bunker Hill took place on June 17, 1775. At the time, the American army occupied the area from Cambridge to the Mystic River. American troops gathered in Cambridge Common on the evening of June 16, 1775, and set out for Bunker Hill. Upon reaching Bunker Hill, however, officers decided to move to Breed's Hill, a smaller hill closer to Boston. Given this excerpt from the chapter, which of the following graphic organizers would best promote students' awareness of the chapter's text structure? A. outline B. Venn diagram C. timeline D. semantic map

Correct Response: C. The text structure of this passage is chronological, focusing on when troop movements took place leading up to the battle. Timelines are the most effective form of graphic organizer for conveying a chronology of events. Options A, B, and D are incorrect because these types of graphic organizer are better suited to other types of passages or purposes. Outlines (A) and semantic maps (D) are best used with passages that are organized into different sections or categories of related content, while Venn diagrams (B) visually display similarities and differences between two or more related topics.

Which of the following principles is best illustrated by the words watched, wanted, and warned? A. Spelling is often the best predictor of the pronunciation of a suffix. B. Open syllables are usually pronounced with a long vowel sound. C. The spelling of a suffix is often more reliable than its pronunciation. D. The second letter of a consonant blend is usually pronounced as the onset of the following syllable.

Correct Response: C. The words listed all contain the regular past-tense inflection -ed. The ending is spelled the same way in all three cases, but the ending is pronounced differently in each word. In the word watched, the -ed ending is pronounced /t/. In the word wanted, the -ed ending is pronounced [ǝd]. In the word warned, the -ed ending is pronounced /d/. Option A is incorrect because the three words illustrate that a single spelling of the ending can be pronounced three different ways. Option B is incorrect because none of the words contain an open syllable and thus do not illustrate characteristics of open syllables. Option D is incorrect because the syllabication guideline given is relevant to only one of the words (the word wanted).

A fourth-grade teacher is planning a lesson focused on promoting students' recognition of distinguishing features of prose, poetry, and drama. The teacher plans to begin the lesson by having students follow along as the teacher reads aloud three short passages—a chapter from a novel, a narrative poem, and a scene from a play. Afterward, the teacher plans to lead a whole-class discussion about the passages. Which of the following post- reading activities would be most effective in helping the students prepare for the discussion and achieve the lesson's objective? A. having students meet in small groups of four or five to consider the question, "Which of the three passages affected you most powerfully?" B. asking students to take notes in their literature journals while they reread the three passages silently and then review their notes before the discussion C. having pairs of students use a graphic organizer to comp

Correct Response: C. This activity helps students develop and organize their observations from the first part of the lesson into comparisons that will be important to the upcoming discussion: identifying similarities and differences in the genres along key dimensions, the major elements of a story. The graphic organizer scaffolds that focus. Grouping students in pairs helps ensure every student participates in the activity, benefiting from the opportunity to expand their ideas in conversation with a partner, and practice making comparisons using content-specific vocabulary. Options A and D are incorrect because these questions focus on personal responses to a single piece and not on comparing distinguishing features of the genres. Option B is incorrect because having students take notes while rereading each piece does not encourage them to compare the pieces nor attend to features of a single piece that are key to such a comparison. The activity lacks the structure to focus students on the learning goal and the upcoming discussion.

Which of the following sets of words would be most effective to use when introducing students to the concept of structural/morphemic analysis? A. late, great, wait, eight B. afraid, obtain, explain, remain C. swim, swims, swam, swum D. pretest, retest, tested, testing

Correct Response: D. In the context of reading, structural analysis is the process of recognizing the morphemic structure of words. Typically, structural analysis is introduced to students in the early elementary grades as a strategy for identifying words with inflectional endings and common derivational prefixes and suffixes that are in students' oral vocabulary. The list in Option D features a phonically regular base word and affixes that are appropriate for beginning readers. The words pretest and retest both include a common prefix, while tested and testing include inflectional endings. Option A is incorrect because the words in this list contain only one morpheme and therefore are not useful for demonstrating structural analysis. Option B is incorrect because the list features words that contain prefixes and roots that are inappropriate for beginning instruction in structural analysis. Option C is incorrect because this list features a base word (swim) with irregular past tense forms (swam, swum). Such forms are inappropriate for introducing beginning readers to structural/morphemic analysis because the morpheme for past tense is indicated by a spelling change rather than by the more typical addition of an inflectional ending.

A third-grade class that includes several English learners is preparing to read a text about the life cycles of various organisms (e.g., plants, mammals, reptiles). Which of the following teaching strategies would be most effective in promoting the English learners' comprehension of the text? A. having the students look up unknown English words using bilingual dictionaries and then make vocabulary lists in both languages B. pairing English learners with students whose home language is English and asking the latter to explain any unknown vocabulary C. activating the students' prior knowledge about the topic and providing visual aids such as illustrations to clarify new vocabulary D. giving the students a list of new vocabulary with definitions and having them try to construct original sentences using the words

Correct Response: C. Two of the most effective instructional strategies for promoting English learners' comprehension of a content-area text are activating the students' prior knowledge of the text's content and contextualizing new concepts and vocabulary through the use of visuals, demonstrations, multimedia presentations, and/or hands-on activities. To contextualize new vocabulary related to plants and animals, the use of illustrations and photographs would be the most effective (and efficient) strategy to use and would also activate any prior knowledge the students had regarding the topic. The strategies described in Options A, B, and D are not likely to be particularly effective in promoting English learners' comprehension of the text because they neither build nor activate background knowledge related to the topic, and they do not help contextualize new vocabulary for learners.

A sixth-grade teacher gives students several essays that present contrasting opinions on a current social issue. The teacher then asks students to consider the following questions as they read the texts. 1. What is the author's opinion on the issue? 2. How might the author's background influence the opinion? 3. What evidence does the author use to support the opinion? These questions support students' reading comprehension primarily by prompting them to: A. monitor comprehension of informational texts. B. identify the theme in expository texts. C. draw inferences from informational texts. D. analyze points of view in expository texts.

Correct Response: D. An opinion piece is a type of essay or argument in which an author takes a stand on an issue and tries to persuade readers by presenting supporting evidence. The teacher's questions are designed to help students analyze the author's arguments by identifying an author's opinion, evaluating how effectively the author supports an argument justifying that opinion, and considering factors that may have influenced the author's opinion. Option A is incorrect because, while students may monitor their comprehension of the texts, comprehension monitoring is not explicitly taught or reinforced by the questions. Similarly, the skills described in Options B and C may be secondary outcomes of this activity but are not the primary goal.

Use the information below to answer the two questions that follow. A kindergarten teacher regularly elicits oral retellings of stories children have listened to or read as a way to assess their understanding of narrative text structures. The retell protocol the teacher uses has a child retell the story to a stuffed animal, named Storalee, as the teacher records notes and checks off story components. The teacher starts with the prompt, "Tell our friend Storalee the whole story because she has not heard it before." The teacher rates each child's understanding of the text's characters, setting, events, and relationships according to standards-based rubric descriptors for story elements Which of the following strategies would be most appropriate for the teacher to use to differentiate the retelling assessment for children who are advanced readers? A. substituting literal comprehension questions for the oral retelling B.

Correct Response: D. Asking advanced readers to elaborate on specific story elements in their retelling prompts them to use higher-order thinking skills (e.g., inferencing) during the assessment, which is an appropriate differentiation strategy for advanced learners. For example, asking the children to elaborate on how a character responds to a story event (e.g., "How do you think the character feels when...") encourages them to make inferences by combining their background knowledge about stories, characters, situations, and feelings with evidence in the text to elaborate on things that may not be explicitly stated in the text. Option A is incorrect because asking literal questions can serve to limit children's retelling to the explicitly stated events or details in the story and does not promote higher-order thinking. Similarly, Options B and C are incorrect because summarizing an informational text (B) or recording the children's retelling (C) does not engage the children in higher-order thinking.

Early in the school year, a fifth-grade teacher analyzes the results of a developmental spelling survey to identify students' strengths and needs as spellers. Using this information, the teacher plans whole-group instruction for spelling elements and patterns the majority of students need to learn (e.g., fifth-grade-level prefixes and suffixes). However, some students have not yet mastered earlier spelling elements or patterns, while other students are ready to learn elements that are beyond the scope of the fifth- grade spelling curriculum. Which of the following approaches to spelling instruction would best address this diversity of skills? A. expanding the number of spelling elements addressed in whole-group instruction B. creating individual spelling lists for each student in the class C. providing differentiated spelling homework to the students who vary from the norm D. planning differentiated instruction using

Correct Response: D. In differentiated instruction, a teacher uses individualized and small-group instruction to align the content, intensity, and pacing of instruction in a particular area (e.g., spelling) with the strengths and needs of the students. Differentiated instruction would allow the teacher in this scenario to instruct students according to their current spelling strengths and needs, as indicated by the assessment data. Meanwhile, flexible grouping means that the teacher continually monitors individual student's progress and changes the makeup of the groups as students' strengths and needs change. Option A is incorrect because this whole-group instructional approach does not take into account that some students would not yet be ready to learn certain spelling elements, while other students would have already mastered many of the elements. This would reduce the opportunities for many students to benefit from instruction. Option B is incorrect because it reflects an outdated and ineffective approach to spelling instruction, in which students memorize lists of specific, unrelated words each week instead of learning spelling skills that are generalizable to many words. Teaching generalizable spelling skills promotes students' development of orthographic knowledge that supports and reinforces both reading and spelling development, whereas memorizing random words has little benefit. Option C is incorrect because homework would not provide most students with the explicit instruction and teacher-guided practice they need to advance their spelling skills.

In the years since the report by the National Reading Panel (2000) was published, evidence-based research has conclusively documented that which of the following phonics approaches is most effective in promoting beginning readers' reading and spelling development? A. teaching students common phonograms or word families that share the same orthographic rime (i.e., ending letter sequence with a common pronunciation) B. introducing students to printed words that are likely to be in their oral vocabularies to support them in making effective guesses based on a text's context (e.g., the pictures) C. providing embedded phonics or "phonics in context," in which the teacher primarily offers support reading a word when a student cannot read a word in connected text D. focusing on grapheme-phoneme correspondences, in which students are taught explicitly to sound out each letter or letter combination in a word and blend the le

Correct Response: D. Option D is correct because evidence indicates that a synthetic phonics approach to decoding instruction produces better phonemic awareness, reading, and spelling outcomes for beginning readers. Furthermore, explicit, systematic phonics instruction, as described in Option D, facilitates the development of more advanced code-based skills (Brady, 2020). Students not only develop skill in decoding specific words but also develop orthographic knowledge that facilitates more rapid decoding of unfamiliar words they will encounter in other texts (Ehri, 2014). Option A is incorrect because the approach described does not promote students' attention to decoding all the letter-sounds in a word, which is essential to developing more rapid and accurate word recognition over time. Option B is incorrect because guessing words based on context is an inefficient and inaccurate strategy for recognizing words. In addition, it does not prepare students to read texts at higher grade levels, which will include fewer and fewer pictures. Option C is incorrect because embedded phonics instruction is not systematic and therefore not supported by evidence as an effective approach to promoting reading and spelling development among beginning readers.

Which of the following statements provides the best rationale for incorporating spelling instruction into a first-grade reading program? A. Spelling instruction promotes phonological sensitivity by teaching students to break words into onsets and rimes and recognize common phonograms. B. Spelling instruction streamlines the reading process by focusing on a finite set of orthographic guidelines, which accelerates students' reading development. C. Spelling instruction facilitates students' vocabulary development by introducing them to new grade-level academic words throughout the school year. D. Spelling instruction reinforces students' knowledge of phonics patterns, which supports their development of automaticity and ability to construct meaning while reading.

Correct Response: D. Option D is correct because evidence indicates that the development of spelling and phonics skills is reciprocal; when instruction in these two skill areas is aligned, development of each skill area is reinforced and enhanced. Thus, learning the spelling patterns of words reinforces phonics learning and supports development of automatic word recognition. Option A is incorrect because the phonological awareness skill of onset-rime segmentation precedes and contributes to students' spelling development, not the reverse as stated in Option A. Option B is incorrect because spelling instruction does not focus on a finite set of orthographic guidelines. In addition, focusing on a finite set of orthographic guidelines would not streamline the reading process. Option C is incorrect because most words targeted in spelling instruction are not new to students but already part of their oral vocabulary.

By halfway through the school year, a majority of students in the class are making good progress reading the poems with fluency. However, a handful of students still read the poems haltingly, word by word, and ignore punctuation. Which of the following explicit, evidenced-based strategies would best transition the students away from word-by-word reading during the daily poetry activity? A. shifting the students from poetry to a broader range of narrative and informational texts B. establishing a weekly poetry performance to motivate the students to read with more expression C. building the students' background knowledge with respect to each poem's theme and literary device D. adding phrase-cues to the students' poetry booklets and modeling how to read aloud in phrases

Correct Response: D. Option D is correct because phrase-cueing both facilitates text comprehension and scaffolds appropriate phrasing, an important component of prosody. Phrase cues are visual marks added to a printed passage at phrase boundaries to help the reader recognize meaningful chunks of text and guide expressive phrasing. Such scaffolded support reinforces the prosodic modeling provided by the teacher or a volunteer adult reader. Option A is incorrect because simply changing the genre of texts would not provide the students with additional support in prosodic reading. Option B is incorrect because, while poetry performance might be motivating to students, such motivation does not provide the explicit, evidence-based instruction required to support students who have not yet developed the ability to read prosodically. Option C is incorrect because building the students' background knowledge related to a poem's theme and literary device focuses on the development of literary analysis skills rather than prosodic reading skills.

A fourth-grade student who reads on grade level and consistently performs well on spelling tests that are part of weekly word study activities often misspells the same words, and other familiar words, in everyday writings. The following table shows examples of typical errors the student makes on class writing assignments and in informal notes to friends. Target Word. Student Spelling Form. From Split. Spilt Printed. Pinted Dependent. Depedent The students overall spelling performance suggests that the student would benefit most from targeted intervention focused on which of the following foundational skills? A: applying common orthographic rules to inflected words B: sounding out and blending letter-sounds to decode a word C: discriminating between consonant and vowel sounds in words D: segmenting all the phonemes in

Correct Response: D. Option D is correct because the pattern of spelling errors typical in the student's everyday writing—transposing letters (e.g., or in form; li in split) and omitting letters (e.g., r in printed and n in dependent)—coupled with the student's strong performance in formal spelling assessments that include the same words suggest that the student overrelies on memorization for spelling tests. Furthermore, the student's performance suggests that they may not pay attention to the sound sequences of words when encoding in everyday contexts and may possibly lack automaticity in phonemic segmentation skills. Options A, B, and C are incorrect because neither the structure of the sample words nor the student's spelling errors provide evidence that the student has difficulty spelling inflected words (A), blending letter sounds to decode words (B), or discriminating between a word's root morpheme and affixes (C).

A first-grade teacher periodically administers spelling surveys to help assess students' phonics knowledge. The following tables show one student's performance on a spelling survey midway through the school year and again two months later. Distracted Word. Student Spelling. Set. Set Star. Ster Drive. Drive Peach. Pech Turn. Tarn. Join. Joyn Distracted Word. Student Spelling Set Set Star. Star Drive. Drive Peach. Peche Turn. Turn Join. Joyn The student's performance on the second administration of the spelling survey indicates that the student

Correct Response: D. Option D is correct because the patterns of improvement across the assessments suggest that the student has learned two conventional spelling patterns, or syllable types. First, by correctly spelling star and turn in the second Correct Response: D. Option D is correct because the patterns of improvement across the assessments suggest that the student has learned two conventional spelling patterns, or syllable types. First, by correctly spelling star and turn in the second assessment, the student demonstrates progress in spelling words with r-controlled vowels. Second, by using the silent-e syllable type to spell the words drive and peach, the student demonstrates knowledge of the VCe conventional spelling pattern for long vowels. Although the student's spelling of peach in the second assessment is incorrect, it nevertheless demonstrates progress in learning the silent-e syllable type commonly used in English. Options A, B, and C are incorrect because the student shows no change over time in the indicated spelling skills.

A kindergarten teacher engages a small group of children in the following Say It and Move It activity. • The teacher says a two-phoneme word slowly (e.g., ape, bee, day, eat, go, she, toe). • The children slowly repeat the word. • The children move a plain wooden block as they say each phoneme, lining up the two blocks from left to right. Once the children demonstrate mastery of this activity, which of the following strategies would be most appropriate for the teacher to use next to build the children's phonemic awareness? A. writing pairs of words on the board that differ by one phoneme (e.g., ape, cape) and pointing out to the children that the second word contains more phonemes than the first B. exchanging the plain blocks for alphabet letter blocks and then helping the children do the Say It and Move It activity with relevant letter blocks, using pairs of words that have two and three phonemes (e.g., go, g

Correct Response: D. Option D is correct because the strategy aligns with the evidence-based practice of increasing the complexity of an instructional task incrementally. In D, the teacher increases the length of the spoken words in the phonemic awareness task by one phoneme. In the initial Say It and Move It activity described, the stimuli are all words with two phonemes. The task described in option D adds one sound to the beginning or end of the same spoken words, thereby increasing the number of phonemes from two to three. By using pictures in the task, the teacher reinforces the concept that a one phoneme difference also changes the meaning of a word. A and B are incorrect because these options not only add spoken words with three phonemes to the original task, but they dramatically increase the task complexity by changing the task from oral to written and requiring the children to recognize phonics/spelling patterns such as VCe and vowel teams. Whereas the task described in C, generating rhyming words, represents a less complex task along the phonological awareness continuum.

Use the information below to answer the the question: A third-grade teacher uses word matrices as a strategy for teaching structural or morphemic analysis skills. The teacher models for students how to create and use a word matrix to generate related words by combining a base word or root with one or more affixes. Following a word matrix for the base move. re un move s ing ed er ment The teacher supports students in generating the following list of words using this matrix: moves, moving, moved, movement, movements, remove, removes, removing, removed, remover, unmoving, and unmoved Which of the following words that the students generated contains derivational suffix? A: moves B: moving C: moved D: mover

Correct Response: D. Option D is correct because the word mover contains the derivational suffix -er, which, when affixed to a verb, creates a noun with the general meaning "someone who performs the action of the verb" (e.g., a mover is someone who causes something to move). A derivational suffix alters the meaning of a base word and almost always changes the grammatical class of the word (e.g., from verb to noun). Options A, B, and C are incorrect because they have inflectional morphemes that mark person (A) or tense (B and C) in a verb. Inflections are a very limited class of grammatical morphemes that do not change a base word's part of speech or alter its fundamental meaning, but indicate grammatical attributes or functions, such as number, mood, person, tense, or case.

A teacher is working with a group of learners who exhibit the following characteristics. • The students can accurately read single-syllable words that feature closed, open, and silent-e syllables; and vowel-team syllables that make a long-vowel sound. • The students can accurately read multisyllable words that feature closed and open syllables. Which of the following sets of words includes appropriate types of syllables to provide the students with practice applying their knowledge of syllable types to read decodable multisyllable words? A. treadmill, healthy, pleasant B. council, county, avoid C. table, pickle, candle D. reveal, combine, explain

Correct Response: D. Option D is correct because these regular words feature syllables with which the students are familiar. The word reveal includes an open syllable and a basic vowel- team syllable that makes a long vowel sound. The word combine includes a closed syllable and a silent-e syllable. And the word explain includes a closed syllable and a vowel-team syllable that makes a long vowel sound. Options A, B, and C are incorrect because the words in these lists feature phonics elements that the students have not learned yet. Options A and B feature advanced vowel teams. The vowel team in treadmill, healthy, and pleasant (A) makes a short vowel sound, while the vowel teams in council, county, and avoid (B) make diphthongs. Meanwhile, the words in Option C end in a consonant + le syllable, which is a syllable type that the students have not learned yet.

A second-grade student demonstrates automaticity decoding grade-level regular and irregular words. However, the student frequently experiences poor text comprehension with informational texts. Which of the following steps should the teacher take first to promote the student's reading development? A. evaluating the student's ability to apply grade-level phonics and syllabication skills when reading the target texts B. determining the rate of the student's phonological processing when reading the target informational texts C. evaluating the degree to which the student uses syntactic clues when reading the target informational texts D. using questioning to determine the student's vocabulary and background knowledge with respect to the target texts

Correct Response: D. Option D is correct because this explanation identifies comprehension challenges frequently associated with informational texts that are independent of the reader's decoding ability. For a student to comprehend a text during reading, they must be able not only to decode each word but also connect the decoded words to words in their oral vocabulary. Content knowledge and academic vocabulary are important components of the background knowledge demands of informational texts. Research has shown that if a reader does not understand the meaning of at least 90 percent of the words in a text, comprehension breaks down. Therefore, a student with strong decoding skills but limited academic vocabulary and content knowledge related to grade-level informational texts may be able to decode the texts accurately but still not be able to make sense of them. Options A and B are incorrect because the student in the scenario already demonstrates automaticity decoding grade-level regular and irregular words, so phonologically based deficits are not likely causing the student's comprehension difficulties. Likewise, Option C is incorrect because readers use syntactic clues as a strategy for determining a word's grammatical function in a sentence. For example, this can be helpful for verifying the meaning and pronunciation of a multiple-meaning word in a text. However, a weakness in interpreting syntactic clues is not likely to be the primary cause of the student's consistent comprehension difficulties with informational texts.

Which of the following statements best describes the relationship between word decoding and reading comprehension in a beginning reader's development? A. Decoding skills and reading comprehension tend to develop independently of one another. B. Development of decoding skills has little effect on the development of reading fluency or reading comprehension. C. Reading comprehension contributes to and directly facilitates the development of decoding skills. D. Decoding skills are essential for the development of reading fluency to support reading comprehension.

Correct Response: D. Option D is correct because this statement reflects decades of evidence related to the development of skilled reading. According to convergent research, accurate, automatic decoding skills are foundational to the development of reading fluency, which in turn is essential to support reading comprehension. Without fluent reading skills, readers do not have sufficient cognitive resources to attend to the meaning of a text. Fluency is not only essential for supporting text comprehension among beginning readers; it continues to be crucial for supporting reading comprehension as students encounter increasingly complex texts in the elementary grades and beyond. Options A, B, and C are incorrect because these statements are not supported by convergent, evidence-based research on beginning reading development.

A second-grade class includes several students who are developing-level (intermediate) English learners. The teacher is planning to use a whole-class read-aloud to provide instruction in making inferences when reading informational text. Which of the following differentiation strategies would best support students with diverse language abilities in making text-based inferences? A. inviting the English learners to read the passages that will be used in the lesson in advance and to refer back to the printed copies of the passages during the lesson B. offering the English learners alternative, literal comprehension-building activities (e.g., "Fill in the blank: The authors said ________ . The text was mostly about ________.") C. collecting a variety of reference materials for the classroom, and making them freely available to the English learners as needed during instruction D. providing the English learners with senten

Correct Response: D. Readers make text-based inferences by combining evidence from the text with relevant background knowledge. Sentence frames like the ones described provide English learners with a language scaffold that supports making inferences based on evidence and background knowledge. Not only does the sentence frame divide the skill into a series of simple steps, but it also makes the linguistic task more accessible for the students as they discuss or write about the content in the passages. According to evidence, English learners benefit from literacy instruction that incorporates explicit language support to complete academic language tasks. Options A and C are incorrect because, while prereading the passages (A) or being able to use reference materials (C) may be useful if the students need support in understanding some words in the passage, these strategies do not support language development related to the target skill, making inferences during reading. Option B is incorrect because this strategy does not provide the English learners with support in inferential comprehension. All students should receive differentiated instruction that makes grade-level instruction accessible.

A third-grade teacher observes that students who can read aloud fluently also demonstrate greater comprehension of expository texts. The best explanation for this is that fluent readers: A. possess a self-awareness that allows them to use metacognitive skills efficiently. B. have already developed the base of background knowledge typically covered by textbooks. C. have well-developed skills for decoding any level of text word by word. D. are able to focus their full attention and cognitive resources on the meaning of a text.

Correct Response: D. Research has shown that fluent readers have higher levels of comprehension than readers who lack fluency. By the third grade, fluent readers have developed automatic word recognition, which allows them to focus on the meaning of what they are reading rather than expending significant cognitive resources on decoding words. Options A, B, and C are incorrect because achieving reading fluency does not necessarily mean that students know how to use metacognitive skills (A) or that they have already developed background knowledge at the level of a textbook (B). Having fluency also does not mean that a third-grade student is skilled enough to decode any level of text (C).

Use the information below to answer the three questions that follow. A third-grade class includes students with delays in foundational reading skills. Two students also have delays in language expression and comprehension. The teacher is considering ways to best support the students' reading development. The teacher would also like to provide appropriate supports for the students during the planned biweekly whole-class close-reading routine, in which the teacher will engage the students in reading and rereading a variety of complex literary and informational passages. Which of the following strategies for differentiating collaborative text-based discussions for the students with language-expression challenges would best enhance their participation in the text-based discussions and their overall language development? A. having the students discuss texts with each other instead of in a larger group B. pairing the stud

Correct Response: D. Students with language-expression challenges often have difficulty with academic language that utilizes conjunctions to connect ideas. Language frames provide a supportive syntactic structure for making a complex statement (e.g., conveying evidence for a response, making a causal statement, providing a reason for one's thinking). Providing the students with appropriate sentence or language frames would support their participation in the text-based discussions, as well as provide them with practice engaging in common academic- language tasks, thus promoting both their literacy and language development. Options A, B, and C are incorrect because they do not address the students' underlying language-expression challenges or provide them with opportunities to produce academic language on their own with appropriate scaffolding.

A second grade class includes several expanding level (advanced) English learners. The teacher typically preview a variety of texts to help select appropriate passage for English language arts instruction. Following is an excerpt from one of the texts the teacher is considering. Edward's and Jo's eyes met. Edward blinked. "There was a letter on the table a second ago. Where did it go?" Jo asked. Edward shrugged. "I wouldn't know," he said. "Don't give me that!" Snapped Jo. Edward could see that his older sister was about to blow a fuse. "No need to bite my head off," he said. "It's got to be here somewhere!" As Jo turned around to scan the room. Edward quickly took the letter from his jacket pocket and chucked it under the table. The English learners are most likely to need support with which of the following comprehension challenges in this passage? A. decoding irregularly spel

Correct Response: D. The passage contains many instances of words and phrases used idiomatically to convey important information about the characters' interaction (e.g., "eyes met," "I wouldn't know," "snapped Jo," "about to blow a fuse"). While English learners with expanding- level proficiency would have been explicitly taught grade-level academic language found in literary texts, they are less likely to have been taught or even exposed to less common idiomatic English phrases and slang. Options A and C are incorrect because they identify grade-level reading and language skills that should not be too difficult for a second-grade English learner with expanding- level English language proficiency. Would and know (A) are first- and second-grade high- frequency words; the words blinked and shrugged (C) are typical of a grade-level literary text. Option B is incorrect because, while applying knowledge of print concepts associated with dialogue is not systematically studied until third grade, this skill is not likely to affect students who are English learners more than their peers whose home language is English. Furthermore, other clues in this passage make the dialogue clear (e.g., "Jo asked," "he said").

A fourth-grade class will be comparing the treatment of a similar theme, "family ties," in stories from different countries. After students read the stories independently, the teacher plans to lead close rereadings focused on the influence of culture on the target theme. Which of the following teaching preparation strategies would be most essential and effective for achieving the goals of this lesson? A. dividing the stories into coherent parts (e.g., beginning, middle, end) and composing a summary of key ideas or information conveyed in each section B. identifying passages key to comprehending the basic plot of each story and particular words or phrases the authors use that might be unfamiliar to students C. defining the key story elements in each story (e.g., time, place, characters) and identifying similarities and differences between these aspects of the stories D. identifying passages in each story that are esse

Correct Response: D. This aspect of the teacher's preparation focuses on identifying specific words or passages in the text that are relevant to the lesson's learning goals and appropriate focal points for discussion. Close rereadings support a text-based analysis of details and an author's use of craft (e.g., language choices, literary devices, text structures, narrative voice) to convey an idea or theme. Options A, B, and C are incorrect because, while identifying these textual elements relevant to students' literal comprehension of the stories may be part of preparing for any close rereading activity, this kind of preparation is not directly relevant to an analysis of the role of culture in the treatment of a theme. These strategies focus on the overall arc of the story or the plot (A and B), unfamiliar vocabulary (B), and story elements that may vary across the stories (C) but that do not necessarily relate to culture.

A fifth grade teacher plans to use the passage below in a lesson focused on analyzing literacy texts. Zander told me it didn't matter what the exact rules were, or what was the "fair" according to Coach. What mattered most was the team winning against Sagagmore and advancing to the next level. "So, okay? You in?" He asked me. I just looked at him without saying a word. I like Zander. He's funny, smart, popular — everything I'm not. So I wanted to say. Sure, let's do it. But I kept seeing Coach's face in my mind, like he was looking right into my eye. "I don't know" I said slowly. Zander's eyes narrowed and his mouth firmly shut. Oh, great, I thought. Now I'll have NO friends at school. The teacher is planning text-based questions to use in a post-reading discussion about the passage. Which of the following organizing questions would most effectively prompt students' higher-order analy

Correct Response: D. This question prompts students to analyze the central conflict faced by the narrator (main character) in the passage: whether to live up to the values that govern their relationship with Coach or, instead, compromise these values to help a popular—but unprincipled—peer, Zander. To answer the question in Option D, students must infer the nature of the two relationships and then compare and contrast these relationships, a higher-order analysis that clarifies what is at stake for the narrator in this scene. Options A and B are incorrect because these questions target students' literal comprehension of the passage. Students can answer the questions by recalling explicitly stated details from the text or by scanning the text without using higher-order literary analysis skills. Option C is incorrect because the question is not relevant to the passage shown. A plot is the sequence of major events in a narrative. In the excerpt, dialogue is minimal, and does not advance the plot.

Use the information below to answer the two questions that follow. A kindergarten teacher regularly elicits oral retellings of stories children have listened to or read as a way to assess their understanding of narrative text structures. The retell protocol the teacher uses has a child retell the story to a stuffed animal, named Storalee, as the teacher records notes and checks off story components. The teacher starts with the prompt, "Tell our friend Storalee the whole story because she has not heard it before." The teacher rates each child's understanding of the text's characters, setting, events, and relationships according to standards-based rubric descriptors for story elements Which of the following components should be included in the story elements rubric to ensure that the assessment will provide information about children's understanding of the cause-and-effect relationships found in narrative texts? A. as

Correct Response: D. When children use causal conjunctions, they demonstrate that they understand and can express a causal relationship supported by evidence or a reason (e.g., "She did X because...," "X didn't work, so she did Y."). If children do not explicitly state connections between characters and events, the teacher cannot assume the children understand the causal relationships in the story. Option A is incorrect because details about events do not necessarily demonstrate an understanding about what caused the event or how it affects the characters in the story. Option B is incorrect because the use of sequence words provides evidence of children's understanding of the chronology or sequence of events in the story but does not provide evidence of their understanding of causal relationships between characters and events in the story. Option C is incorrect because questions direct children's responses. If the teacher asks a question that requires children to think about causality, then the teacher is directing the children to make causal connections. Questioning can be an effective instructional strategy for supporting students in learning how to make connections between characters and events; however, this is not an effective assessment strategy for determining whether children understand the causal relationships in the story on their own, without teacher prompting.

Use the information below to answer the question that follows A teacher poses the following question to fourth-grade students What words can you think of that have the word "act" in them? Using student responses, the teacher creates the following web on the board To the left: Acts, acted, acting, actor, active, action, activity, actual, actually, activate, activism, activation On top: React, entact, overact, interact On bottom: Play act To the right: Inactive, deactivate, transaction, enactment The technique is likely to be helpful for enhancing the students' awareness of: A: morphemic structure B: compound words C: syllable patterns D: Greek roots

The web shows a circle with the base word act in the center and connected to four word lists that are categorized according to different morphological processes—clockwise from left, adding suffixes or inflectional endings, adding prefixes, adding both, and creating compound words. Thus, the chart illustrates how to categorize multimorphemic words according to morpheme type and how these types of morphemes are combined, which promotes students' awareness of the morphemic structure of words. Option B is incorrect because only a single compound word is presented in the web, which is insufficient to illustrate and promote understanding of this word type. Option C is incorrect because syllable patterns are not the focus of analysis. Option D is incorrect because the only root word in the web, the base word act, is derived from the Latin word actus and is not a Greek root.


Set pelajaran terkait

Marketing 300 Exam 1 - Hapke - Chapter 2

View Set

Health Assessments: Cranial Nerves + Review Questions

View Set

Module 00102 Intro to Construction Math

View Set

Chapter One - Personal Property vs. Real Property

View Set

Fetal Environment and Maternal Complications

View Set

Chapter 16: America's Gilded Age, 1870—1890

View Set

Earthquakes in Your Backyard Final

View Set